Vous êtes sur la page 1sur 57

CONTRACTS OUTLINE

I) BASIS FOR RECOGNIZING AN ENFORCEABLE OBLIGATION

A) The Meaning of “Enforce”: A contract is a promise or set of promises for the


breach of which the law gives a remedy, or the performance of which the law in
some way recognizes as a duty (Restatement of Contracts – Second, Sec. 1); a
contract is an agreement or promise which is enforceable by law.

1) Two Basic Assumptions in Enforcement of Promises:


(a) The law is concerned mainly with relief of the promisee to redress
breach and not the punishment of promisors to compel performance.

(b) Relief granted to the aggrieved promisee should generally protect the
promisee’s expectation by attempting to put the promisee in the
position in which he would have been had the promise been
performed.

Ex. U.S. Naval Institute v. Charter Communications; Naval was only entitled to
the profits that it reasonably could have gained had the contract been
honored, they were not allowed a portion of Charter’s profits.

2) Economics of Remedies: Sometimes a breach of contract would be


economically efficient and beneficial for the breaching party because the
amount they would have to pay for the breach would be less than the
economic gain. This is called an efficient breach.

3) Three Approaches to Damages for Breach of Contract:


(a) Expectancy: restoring the value of what the injured party could have
expected had the contract (K) been honored. This is the usual method
for determining damages; other methods are used in special cases.
(Restatement, Sec. 347).
(b) Restitution: restoring the monetary benefit gained by the breaching
party. Ex. Fee paid to a doctor. (Restatement, Sec. 371).
(c) Reliance: placing the injured party back into a position they would
have been had they never relied on the K. (Restatement, Sec. 349).

Ex. Sullivan v. O’Connor; the court used a both an expectancy and reliance
method, holding that Sullivan could recover for the worsening of her
condition (which would never have happened had she not relied on the
contract) as well as the pain and suffering of a third operation (which was
not expected in the original agreement).

4) Third Assumption in Enforcement of Promises: the appropriate form of relief


is substitutional, usually through monetary damages, rather than specific, such
as a court requiring the promisor to perform his promise.
(a) However, courts have allowed specific performance in contracts which
involve land and agricultural commodities.

5) Punitive damages are not traditionally granted for breach of K.


However, courts have granted punitive damages for tortuous conduct that
is sufficiently outrageous. Ex. Broken insurance contracts and bad
business relations.

B) Consideration as a Basis for Recognizing an Enforceable Obligation


1) Fundamentals of Consideration: Consideration (C) is only one basis for
enforcement; consideration is neither necessary nor sufficient to enforce a K.
(a) Consideration Defined: An agreement is said to have consideration when
there is a bargained for exchange and either there is a benefit to the
promisor or a detriment to the promisee. (Restatement 2d, Sec. 71).
(a) A performance or a return promise must be bargained for: it is sought
by the promisor in exchange for his promise and it is given by the
promisee in exchange for that promise.
(b) Benefit to the promisor OR
(c) Detriment to the promisee
(b) Ex. Hamer v. Sidway; the court held that to establish C it was enough
that the nephew had forborne a legal right (the right to drink and
smoke), it was not necessary that there be both a benefit to the
promisor and a detriment to the promisee, only that one or the other
had occurred.
(c) Gratuitous promises:
(d) Ex. Feige v. Boehm, 1956: A forbearance based upon a false question
which was believed to be true at the time is sufficient consideration for
promise.

2) Exchange; Action in the Past: Generally, actions done in the past and moral
obligations are not sufficient C for enforcing a promise.
(a) Restatement 2d of Contracts, Sec. 86: a promise made in recognition of a
benefit previously received by the promisor from the promisee is binding
to the extent necessary to prevent injustice.
(b) Ex. Feinberg v. Pfeiffer, 1959: the past services of an employee are not
C for a promise to pay retirement benefits.
(i) There was no mutual exchange; company did not ask for her services
in exchange for the benefits.
(ii) This was simply a gratuitous gesture on the part of the company.
(c) Moral Obligations, Ex. Mills v. Wyman, 1825: father promised to pay for
the medical expenses incurred after Mills had cared for his son, but failed
to fulfill the promise; A moral obligation to fulfill a promise is not C for
a promise to pay the medical expenses incurred by Mills in caring for
Wyman’s son.
(d) Exception to the rule, Ex. Webb v. McGowin, 1935: McGowin promises
to pay Webb money for the rest of his life after Webb seriously injures
himself saving McGowin’s life. A moral obligation to fulfill a promise is
C for a promise where there is a substantial and direct material
benefit to the promisor.
(i) Bargained for exchange is not the end all be all of consideration
(ii) There was a great benefit to McGowin (life saved) and a great
detriment to Webb (crippled for life)
(iii) Thus, this case was substantially different from the Mills case
(iv)Think of this ‘moral consideration’ as a very special exception to the
rules of consideration.

3) Promises as Consideration
(a) What constitutes a promise? A promise that is bargained for is
consideration if the promised performance would be consideration.
(i) One must determine whether any performance at all has been promised
or whether only the illusion of performance has been held out.
(b) Ask yourself: What is the promise at issue? Is there a benefit to the
promisor or detriment to the promise? Is there consideration for the
promise at issue?
(c) A promise can be consideration for the promise at issue, but the promise
must be real and meaningful. Ex. Strong v. Sheffield; Strong’s promise
not to collect debt immediately is not definite enough to constitute a
real promise; it is illusory.
(d) Real Estate K’s; K’s which include satisfactory clauses made in good
faith are okay, NOT illusory. Ex. Mattei v. Hopper
(e) K’s for Sale of Goods; K’s which include requirements clauses are okay
in commercial situations as long as reasonableness and good faith are
adhered to, even though the promise is made to buy “as much as they
need,” NOT illusory. Ex. Eastern v. Gulf
(i) UCC governs only K’s for the sale of goods, not all commercial
transactions
(f) Implied promises: A promise may be implied by the circumstances or
actions of individuals though it is never explicitly expressed. Ex. Wood v.
Lucy; The court held that there was an implied promise on the part of
Wood, a professional, to give his best efforts in marketing Lucy’s
name.
(i) Promise of giving your best efforts is sufficient C for a promise.

4) The Requirement of Bargain


(a) There must be a bargained for exchange in order for a promise to be
supported by consideration. Ex. Kirksey v. Kirksey; Court held that
plaintiff’s selling of her home and moving of her family was not
bargained for and thus cannot be consideration for defendant’s
promise to provide a house and land.
(b) Employment K’s; generally, there must be a bargain, but in an at-will
employment situation, an employee’s promise given in exchange for
continued employment can constitute consideration for that promise, even
though there was never really any bargain between the employee and
employer. Ex. CAB v. Ingram.
(c) Employment Handbooks: Some jurisdictions have held that the policies in
an employment handbook are enforceable; however, Bankey v. Storer held
that these handbooks only help to establish workplace policy, and are
solely a benefit to the employer; once they are changed that benefit ceases
to exist and so does the rationale for enforcing them.
(d) Rewards: Generally promised rewards can be enforceable if the person
accomplishing the task did so in the knowledge of the reward. If they did
not know of the reward, the reward is not enforceable.

C) Alternative Bases for Recognizing an Enforceable Obligation: Obligation in


the Absence of Consideration
1) Reliance as an Alternative (Non-Contractual) Basis for Recognizing an
Enforceable Obligation: Enforceability of promises in the absence of
consideration
(a) Elements of promissory estoppel (reliance):
(i) Pr should reasonably expect to cause Pe to rely on the promise;
(ii) Pe does indeed rely to his/her detriment;
(iii) Enforcement is necessary in order to avoid injustice.
(b) Rickets v. Scothorn: grandfather’s promise to pay money to
granddaughter is enforceable because he implied that she would no
longer have to work, and she did indeed quit her job.
(i) There is a relationship of trust between family members that would
make non-enforcement unjust
(ii) Note that there is no exchange or bargain here, but there is an
implication by the grandfather.
(c) Feinberg v. Pfeiffer: company’s promise to pay Feinberg pension
benefits upon retirement is enforceable because Feinberg retired
earlier than she might have had the promise not existed.
(i) You have to actually do something in reliance, such as retire early
(ii) This is a believable story: it makes sense that an older lady might retire
early because of a promise to pay pension benefits.
(d) Restatement 2d Sec. 90
(e) Cohen v. Cowles: newspaper’s promise to keep Cohen’s identity
confidential is enforceable because Cohen, in reliance upon the
promise, was fired from his job when the promise wasn’t kept.
(i) The fact that there was an established custom or ethic for keeping the
confidentiality of sources, made the breaking of that promise
sufficiently unjust for the purposes of reliance doctrine
(ii) The custom in itself is not sufficient for a reliance remedy, but it helps
to support the contention that there was a particular injustice done
which required a remedy
(f) D&G Stout v. Bacardi: Bacardi knew of the negotiations between Stout
and National and could have reasonably expected Stout to rely on their
promise to stay with Stout; Stout relied to their detriment by rejecting
National’s lucrative offer; context was particularly unjust to warrant a
reliance remedy.
(i) The decision was specific to the facts; court was not trying to establish
a deadline for when someone can back out of an at-will relationship.
2) Restitution as an Alternative (Non-Contractual) Basis for Recognizing an
Enforceable Obligation; Obligation in the absence of an enforceable promise
(a) Sometimes enforceability can be found in situations in which there is not
an enforceable promise or no promise at all.
(b) A quasi-contract is created (it is a legal fiction) by the court in order to
provide a remedy:
(i) Cotnam v. Wisdom: Surgeon was compensated via a quasi contract
for his attempts to save an unconscious man’s life, even though he
never expressed a promise to pay the surgeon.
(ii) Callano v. Oakwood: Landscape company cannot be compensated
via a quasi-contract because they could not have reasonably
expected pay from the developer, but only from the proposed
buyer who actually ordered the shrubs
1. Oakwood was enriched by the shrubbery, but not unjustly
(iii) Pyeatte v. Pyeatte: Marital promises are generally not
enforceable, but court created a quasi-contract where husband
backed out of a promise to pay for graduate school, in return for
the wife’s unilateral detriment in putting him through law school.

D) Reforming the Doctrine of Consideration


Gratuitous Promises:

I) THE BARGAINING PROCESS

B) Assent
1) Two views in showing mutual assent:
(a) Subjective: trying to figure out the “actual intentions” of the parties when
they made the agreement; trouble in showing actual proof
(b) Objective: looking only at the outward and expressed actions of the party
and determining how a reasonable person would interpret them; problem
in determining what is reasonable
(c) Objective viewpoint has largely won out
2) Lucy v. Zehmer: the Court held that Lucy, through his actions, could have
objectively been seen to assent to the terms of the contract, even though
he privately believed the contract to be a joke.
(i) The outward expressions of the parties showed such a discussion and
negotiation over the wording of the promise that a reasonable person
would have believed that a good faith offer and acceptance had
occurred.
3) Leonard v. PepsiCo: Leonard and PepsiCo could not possibly have
assented to the sale of a Harrier jet when Pepsi’s commercial advertised:
Harrier jet: 7,000,000 Pepsi Points because the commercial was clearly
fanciful and no reasonable person would believe this constituted an offer.
4) Intent to be Bound: parties are generally not said to be bound under certain
circumstances:
(i) Doctor-patient relationships: optimistic statements made about medical
procedures, unless there is a specific promise made (such as Sullivan
v. O’Connor).
(ii) Statements made between intimates or for social purposes
(iii) Promises made between married couples that would be normally
seen as gratuitous in the marriage relationship
5) The idea is to have rules of enforceability that will make it easy enough for
nonlawyers to formulate K’s but not so easy as to hook the casual promisor.
6) “Gentleman’s Agreements”: at time parties will write into their agreements
“letters of intent” which express an intent to come to a deal, but stipulates that
there will be no liability if the deal falls through.
7) “Formal Contract Contemplated”: sometimes business K’s will agree on
essential terms of a contract and leave the details to be worked out. Whether
these K’s are enforceable if one party refuses to sign the final document
depends upon:
(i) Whether there is an expressed intent that no K shall exist;
(ii) Whether there has been a partial performance of the K;
(iii) Whether all of the terms have been agreed upon;
(iv)Whether the agreement at issue is the type of K that is usually
committed to writing.

C) Offer: when the offeror assents to give the offeree the power to make a contract.
1) Mutual Assent: Offer + Acceptance = K. This is the model, but it is idealized.
You rarely see a clear offer and clear acceptance that constitutes a binding
contract.
2) What is an offer? An act whereby one person confers upon (assents to) the
other the power to create contractual relations between them.
(i) The offeror creates a situation in which all the offeree has to do is
make a simple manifestation of his assent and he can effectively bind
the offeror to his word.
3) Owen v. Tunison: Tunison’s statement “I would not be able to sell unless I
were to receive $16,000 in cash,” did not constitute an offer that could be
accepted by Owen.
(i) There needed to be more specific terms (definiteness) for Tunison’s
statement to be considered an offer
(ii) A reasonable person could speculate that Tunison was merely trying to
induce Owen to make a better offer, NOT make an offer himself.
(iii) The fact that they are communicating with letters could serve to
help Owen because it shows that each party had time to reflect on their
responses.
(iv)A better drafting from Tunison would have been: “I will not entertain
an offer for less than $16,000.” That would have made it clear that he
was not making an offer.
4) Harvey v. Facey: Facey’s reply that the lowest price he would sell Bumper
Pen was 900 pounds did not constitute an offer.
(i) Harvey had asked 1) will you sell? 2) what is your lowest price? Facey
only replied to the second question, thus NO OFFER!
5) Fairmount Glass v. Crunden-Martin: the Court held that Fairmount’s price
quotation contained definite terms which made the quotation an offer,
which could not be withdrawn once Crunden accepted the terms.
(i) REMEMBER: Price quotations are generally not considered offers.
(ii) Fairmount’s quotation was interpreted to be an offer because:
1. Usage of “for immediate acceptance” after quoting prices
2. Specificity: Previous letters from Crunden-Martin had
specified the exact amount of product: ten carloads
3. Ten carloads had definite meaning in the industry
4. Fairmount’s quotation of different prices for different sizes
gave Crunden the power to name the quantity of each size.
6) Advertisements
(i) As a general rule an advertisement is not an offer, but rather an
invitation by the seller to the buyer to make an offer to purchase.
(ii) However, where the advertisement offers a specific item for a specific
price, under clear and defined terms (clear, definite, and explicit) it can
be considered an offer, the acceptance of which makes a binding
contract.
(iii) Lefkowitz v. Great Minneapolis Store: the ad: “first come, first
served; only 1 black lapin stole; for $1; Saturday only at 9 AM,”
set out clear, defined terms and thus constituted an offer, which
became a K when Lefkowitz accepted by being the first person
there.
1. The ad must be show a specific price for a specific item,
anticipating a specific buyer.
(iv)Leonard v. PepsiCo: even if one could have shown assent, the Pepsi
commercial was not definite enough to constitute an offer because
it specifically reserved the details of the offer to a separate writing,
the Catalog.
1. There was no mention of what steps must be taken to accept
the offer of a Harrier jet.
2. The Catalog made no mention of the Harrier jet; thus, it was
obvious that the jet was used to attract attention and nothing
more.
3. It is similar to a car commercial which shows the sticker price.
You cannot go the dealership and demand the sticker price
advertised and sue if the dealer refuses to sell at the price.
D) Acceptance: exercising (assenting to) the power given by the offeror to create a
binding contract.
1) Rule: there must be some sort of sufficient act (performance) or expression
(promise) which shows a clear intention to “accept” the offer.
2) Rule: The offeror does not always need to be notified of the acceptance in
order for a K to be formed; a K can be formed by the appropriate act or
expression
3) International Filter v. Conroe Gin: the Court held that IF had accepted
Conroe’s offer and thus Conroe could not withdraw afterwards.
(i) IF had stipulated that the agreement would not become a K until
“accepted” by their executive officer; they reserved the power of
“acceptance” for themselves.
(ii) Conroe’s acceptance of the terms in agreeing to buy the water softener
and filter was thus actually only an “offer.”
(iii) When IF’s executive officer signed off on the offer, it was
accepted and became a K.
4) White v. Corlies & Tift: the court held that there was no contract because,
even though White may have begun work on the project, he never showed
an explicit intent to accomplish the specific job offered by Corlies & Tift.
(i) White could have accepted either through a return promise to do the
work or by beginning performance of the work
(ii) He did not return any promise
(iii) He bought wood and began working on it in his home, but, because
he is a carpenter, this does not show an intent to commence this
specific job; he could have been working on any job.
5) Ever-Tite Roofing v. Green: Ever-Tite had “accepted” Green’s offer once
they loaded the trucks and started the long trip to the Green house to re-
shingle their roof. The revocation of the offer once they got there was too
late.
(i) There is no one specific thing that a party must do (nail the first
shingle, load the first pallet), the Court is looking for a sufficient act
which shows a clear intention to “accept” the offer.
6) Notice in Unilateral K’s: Where acceptance is invited by means of a
performance, notification can be dispensed with if the offeror expressly or
impliedly intimates in the offer that notification is not sufficient.
7) Ex. Allied Steel v. Ford Motor Co.: the court held that Allied had
“accepted” Ford’s offer, even though they did not return the
acknowledgement form, because they commenced performance with
Ford’s knowledge, consent and acquiescence.
8) UCC 2-206: the promise of prompt shipment of conforming or non-
conforming goods or the actual prompt shipment of such goods shall be
construed as acceptance, unless the shipment is non-conforming and buyer is
seasonably notified that the shipment is made as an accommodation.
9) Ex. Corinthian v. Lederle; Lederle’s shipment of the 50 vials was not an
“acceptance” because the shipment was non-conforming and it was
shipped as an accommodation to Corinthian. Thus, there was no K.
10) Silence Not Ordinarily Acceptance

E) Termination of the Power of Acceptance


1) There should be some limitation on the power given to the offeree; he
shouldn’t be able to hold it over the offeror for an indefinite period of time.
2) Four ways that the power can be terminated:
(a) Offer can lapse: offer automatically lapses after a “reasonable time”; this
does not mean that the Or must keep the offer open for a “reasonable
time.”
(b) Revoked by the Or: generally, Or can revoke the offer at any time before
the offer is actually “accepted” by the Oe.
(i) Option contracts: Sometimes an Oe will want to keep the offer open,
so Oe will give Or something in exchange for Or promise to keep the
offer open (supported by C). If such a K is created, the offeror must
keep the offer open for the specified period; he cannot revoke.
(ii) Three ways to enforce option K’s:
1. Consideration: give something in exchange for keeping the
offer open.
2. “Firm Offers” made by merchants, which are written and
signed
3. Partial, Commencement of Performance
(iii) Ragosta v. Wilder: Wilder had not created an option K and
could revoke his offer because there was no consideration (he gave
back the $2,000) and Ragosta had not begun to accept the offer.
(c) Death of the Or: an offeree’s power of acceptance is terminated by the
offeror’s death or supervising incapacity.
(i) Death or incapacity of the offeror does not terminate the offeree’s
power of acceptance under an option contract.
(d) Rejection by Oe: the rejection of the offer by the Oe terminates the power
of acceptance so that the Oe cannot thereafter accept the offer
(i) Offer is rejected by appropriately notifying the Or. Ex. Cartoon: “Your
offer’s a crumpled little ball in the middle of my desk!” The offer is
not rejected because the man crumpled the paper that the offer was
written on, it is rejected because he called the offeror and notified him
that he had rejected the offer.
(ii) Mirror Image Rule: Traditional common law doctrine insists that an
acceptance must be on the terms proposed by the offer without the
slightest variation.
1. If there is variation, it is assumed that the offer is rejected and a
counter offer is proposed.
2. This rule applies to service K’s; there are different rules for
sale of goods K’s (UCC 2-207).
(iii) “Last Shot” Rule: this is the natural follow to the “mirror image”
rule; if the transaction is carried out, the terms of the party that made
the last expressed offer or sent the last form before performance began
prevails.
(iv)Hypothetical: 1) MPS Emp. Agcy. form quoted labor rates with non-
solicitation clause; 2) Dresser-Rand sent purchase order for labor,
objecting to non-solicitation clause; 3) For 5 years, MPS sent contracts
w/clause. D-R refused to sign, but paid for labor; 4) D-R hired MPS
employees in violation of clause. Was this a breach of K? No. Because
MPS had performed for 5 years in “acceptance” of D-R’s last counter-
offer (no non-solicitation clause). D-R had the “last shot.”
(e) The Mailbox Rule: when Or seeks acceptance by mail or similar method,
Oe’s acceptance is effective upon dispatch by that method. In other words,
once the Oe put the acceptance in the mail, he is considered to have
“accepted” the offer.
(i) Or’s revocation, however, is effective only on Oe’s receipt (Rest. 2d
42).
(ii) The Or could specify: “Offer is only accepted upon receipt of
acceptance letter.” But, if he does not, the mailbox rule is in effect.
(iii) Effect of rule on revocation and rejection: Because acceptance
is effective on dispatch, one Oe dispatches, it is too late for Or to
revoke and too late for Oe to changer her mind and reject.

F) “Battle of the Forms” and UCC


1) Battle of the Forms: refers to the disparity in fine print terms of standardized
forms created by business firms in the commercial context, and determining
which terms will be controlling when these firms enter into a K.
(i) Firms will have a set of terms and conditions for purchases or sales
printed on the business documents used in exchanges
(ii) The terms are in small print and are often not read by the other party
(iii) One party may sign off on a purchase order and the other on an
acknowledgment form
(iv)They will act as if they have formed a K
(v) However, they may not know it but they are in disagreement over the
terms
(vi)Deciding whether or not they have formed a K and what terms apply is
called the “battle of the forms”
2) UCC 2-207: “expressions of acceptance” may “operate as an acceptance even
though it states terms additional to or different from those offered.”
(i) Sec. 1: A definite and seasonable expression of acceptance or a written
confirmation sent in a reasonable amount of time operates as an
acceptance, unless acceptance is expressly made conditional on assent
to the additional or different terms. If it is expressly conditional, then
other party must express assent to the terms or no K.
(ii) Sec. 2: If there is a K, then additional terms are construed as proposals.
If both parties are merchants, such terms will become part of the K
unless:
1. The offer expressly limits acceptance to the terms of the offer
2. The additional terms materially alter the offer
3. Notification of objection to these terms has already been given
or is given within a reasonable time after notice of them is
received.
(iii) Sec. 3: If no K by the writings, but conduct implies that each
thought there was a K, a K can be created and the terms will be those
that the parties agree upon or are supplementary with the rest of the
UCC.
(iv)REMEMBER: This applies only to sale of goods, and it applies to
ANYONE involved in a sale of goods, NOT just merchants (only a
small portion of Sec. 2 applies only to merchants.
3) 2-207 v. Mirror Image Rule:
(i) Mirror image rule establishes a K and its terms at the same time: the
last offer that was on the table before acceptance stipulates what the
terms will be.
(ii) 2-207 establishes a contract in certain situations even when there is
differing terms on the table and then sets up a process to determine
what the terms will be for the contract.
(iii) 2-207 distinguishes between the formation of a K and the terms of
a K; whereas mirror image rule does this at the same time. 2-207 sets
up a system to determine whether or not there is a K and then sets up a
system to determine what the terms are.
4) Dorton v. Collins & Aikman: the Court remanded the decision over
whether or not Dorton should be bound by an arbitration clause included
in an acknowledgement form, asking the court to determine whether or
not the arbitration clause materially altered the offer.
(i) Court found that there was a K under Sec. 1 of 2-207 because there
was no expressly conditional assent clause
(ii) Because they are both merchants, the arbitration clause becomes a part
of the K, unless it would materially alter the initial offer.
(iii) Material alteration can be determined by factors such as 1) whether
or not Dorton and Collins had discussed the arbitration when the offer
was made; 2) whether or not there is an industry custom regarding
arbitration clauses; or 3) whether or not the arbitration clause would
cause a significantly different outcome.
5) Materiality: terms may be found to materially alter if their inclusion would
result in “surprise or hardship if incorporated without express awareness by
the other party.”
(i) Industry custom can also play a factor
(ii) Materiality may differ from state to state
6) Step Saver v. Wyse Technology: TSL’s disclaimer of any and all
warranties printed on a box-top was not “expressly conditional” to the
assent of Step Saver, and thus a K was formed when Step Saver bought
and used TSL’s program; however, the disclaimer materially altered
their agreement, and so it did not become a part of the agreement.
7) Itoh v. Jordan: Jordan’s terms, including an arbitration clause, were
“expressly conditional” on the assent of Itoh, but Itoh did not assent, so
there was no K under 2-207(1); however, there was a K under 2-207(3)
because they proceeded to act as if a K had been formed. The arbitration
clause was NOT a part of the K however because it was not agreed upon
nor supplementary.
8) Shrinkwrap Terms; ProCD v. Zeidenberg: Court holds that 2-207 does NOT
apply, but finds that Zeidenberg was still bound by the terms expressed
in the license enclosed in ProCD’s shrinkwrapped box.
(i) The terms limited Zeidenberg to using the product only for non-
commercial purposes; a term he disobeyed
(ii) Professor believes that reasoning is flawed; 2-207 should still apply;
doesn’t matter that there is only one form, it is important only that this
is a sale of goods issue.
(iii) It could be argued that the opening of the box (removal of
shrinkwrap) is a form of assent in an “acceptance which is expressly
conditional to the customer’s assent to additional terms,” thus creating
a K under 2-207(1).
(iv)If Zeidenberg really didn’t like the terms, he could have easily taken
the product back to the store; that is not a heavy burden.

G) The Requirement of Definiteness


1) Parties are required to know what was agreed upon and what promise we are
trying to enforce; if the terms of the K are not definite, how can there really be
consideration of mutual assent?
2) Definiteness can be pieced together from preliminary negotiations, prior
communications, or references from external sources; it doesn’t all have to be
in one writing.
3) A definite K should include a specific price and quantity; if either of these
things is missing, the “definiteness” of the K is questionable.
4) UCC 2-305: Under certain circumstances, parties “if they so intend can
conclude a K for sale even though the price is not settled.”
5) Toys v. Burlington Co.: the court held that the K between Toys and
Burlington was “definite” and enforceable even though the use of the
wording: “shall be renegotiated to the then prevailing rate” was
somewhat vague.
(i) The prevailing rate of the time was debatable.
(ii) However, the Court interprets the language more favorably to finding
a K, even though there does seem to be a little vagueness in the terms.
(iii) In this case, the court is willing to find a K, where it is clear that
the parties wanted to be bound by a contract
(iv)There is also something shady about Burlington trying to get out of a
contract that they had drawn up by saying that the terms were too
indefinite.
6) Oglebay Norton Co. v. Armco, Inc.: the court found a shipping K
enforceable even though the terms of the K had broken down and ceased
to provide for an established price; the court ordered specific
performance through 2010 and established mediation where the parties
failed to reach an agreed upon price.
(i) The price was to be determined by one of 2 different methods
depending on the circumstance: 1) prevailing rate (“Skillings”); 2) if
no prevailing rate, then parties must mutually agree on a rate, taking
into consideration the rates being charged for similar transportation.
(ii) However, it came about that there was no published prevailing rate and
the other companies refused to disclose the rates they were charging.
(iii) Court finds a K probably because there is a close, longstanding
relationship between the two parties, AND it is clear they both
intended to be bound.
(iv)UCC 2-305 does NOT apply (not a sale of goods), but court uses it as
a guiding principle.

H) “Precontractual” Liability (or, The Bargaining Process Meets Promissory


Estoppel)
1) Construction Contracts, Mistaken Bids
(i) Usually, offers are revocable unless an option K is created; an offeror
cannot be held liable when an offeree relies to his detriment on
offeror’s offer.
(ii) However in the context of the bidding process in the construction
industry, reliance on an offer is allowable and a general contractor can
enforce a subcontractor’s offer when he relies to his detriment.
(iii) Drennan v. Star Paving Co.: the court held that the Star’s
subcontractor bid was irrevocable on promissory estoppel grounds
and once Drennan received the general contract and formally
accepted Star’s bid (offer), an enforceable K was formed; a
subcontractor bid cannot be revoked.
1. The promise to keep the offer is open is suggested by the
overall industry structure of construction contracting.
2. Drennan stuck their neck out in submitting a bid for the
project; they are going to be bound by their bid if they are
selected; if subcontractors are allowed to back out Drennan
would be hung out to dry. If this were allowed, the whole
contracting structure would fall apart.
3. Star induced Drennan to rely on their promised offer, Drennan
relied to their detriment (they ended up having to hire another
person for more money), and it is sufficiently unjust that a
remedy is required.
4. Star’s bid is NOT accepted when Drennan puts Star down as
the subcontractor on the general bid because if that were the
case Drennan would be bound to use Star even if they did not
get the project, which makes no sense.
(iv)Holman v. Orville Madsen & Sons: the court held that a general
contractor is not bound to use the subcontractor that he listed in
his general bid.
1. Holman argues that there should be symmetry: if the general
contractor can bind a subcontractor to his bid, a subcontractor
should be able to hold the general contractor as well.
2. However, promissory estoppel is not symmetrical: general
contractor stands to lose much more if the subcontractor backs
out than the subcontractor does if the general backs out.
Subcontractor only loses the price of submitting a bid.
3. Affirmative action policy requires flexibility for general
contractors in meeting the right amount of minority workers.
2) When Contract Negotiations Fail
(i) Sometimes parties can be found liable even before a K has been
formed, in the negotiation stage, for certain conduct which causes the
other party to detrimentally rely upon promises made.
(ii) Hoffman v. Red Owl Stores: H wanted a Red Owl franchise; RO
promised franchise for $18,000; RO tells H to take a number of steps
before getting franchise; after doing all the things asked, the deal falls
through; court held that Hoffman is entitled to reliance damages
for the actions that he took in reliance upon RO’s promise to sell
him a franchise, even though no K was ever established.
1. At the beginning of this story Hoffman owned a bakery, and at
the end, after he had relied upon many of Red Owl’s promises,
he was working the night shift in someone else’s bakery.
2. There was no K because there was no bargained for exchange
(mutual assent) and no definiteness: $18,000 for what? Red
Owl seems to lead Hoffman on, saying do these several
different things and then maybe we will give you a franchise.
3. However, the Court finds that Hoffman is entitled to recovery
under a doctrine of promissory estoppel. The court says that
you don’t always need a definite, detailed and clear
promise in order to rely to your detriment.
4. RO induced H to move and sell businesses by promising a
franchise; Hoffman stuck his neck out (relied to his detriment);
and Hoffman was unjustly hurt by the result
5. Reliance damages; Hoffman was awarded:
i. Loss on sale of the bakery
ii. Loss on sale of the small grocery: only the difference
between the fair market value and the price sold; NO
future profits.
iii. $1,000 invested in Chilton lot
iv. House rental in Chilton
v. Moving expenses to Neenah
(iii) Factors in Showing Pre-Contractual Liability:
1. Is the offer definite enough to be reasonably relied upon?
2. Look at the nature of the relationship to see if it was reasonable
to rely on a promise or an offer in this case.
3. Don’t need definiteness in the same way in a reasonable
detrimental reliance case, you can calculate damages by look at
what H lost, NOT expectation damages, reliance damages.

II) THE REQUIREMENT OF A “WRITING”: THE STATUTE OF FRAUDS

A) Review: what do you need to have and enforceable contract?


1) Consideration (Common law)
2) Mutual Assent (Common law)
3) Definiteness (Common law)
4) Sometimes a “Writing” (This is where the Statute of Frauds comes in).

B) Purpose of Statute of Frauds: the intent was to stop someone from fraudulently
asserting or denying a K. Ex. Someone saying: “You agreed to sell me your
house.” The Statute’s response is: “Show me that in writing.”

C) Types of K that require a “writing”:


1) Suretyships: promise to answer for the debt of another; Ex. Strong v.
Sheffield; a promise made by a relative or friend of the primary debtor
(Louisa’s husband in that case) made with the benevolent purpose of enabling
that person to get credit or an extension of credit.
2) One year contracts: agreements not to be performed within a year of their
making
(i) This does not apply to an agreement that is capable of performance
within a year, even if a longer period of performance is probable.
(ii) Employment: The one-year clause does not apply to an oral agreement
for lifetime employment; but it does apply to an oral agreement for
employment for a period longer than a year. (See Pg. 276 for more).
3) Sale or Transfer of Real Estate;
4) UCC 2-201: K for the sale of goods > $500
5) Each state will have other types of K that must be in writing; always look up
the state’s s/f; DON’T GUESS!!

D) Applying the Statute of Frauds:


1) Is a writing required? ie. Is subject matter of this K within this state’s s/f?
2) If so, is there a written agreement or a written memo of the agreement?
3) If so, is the writing “subscribed” (signed) by the party to be charged?
(UCC 2-201(2) has a ‘merchants’ exception)
(i) Why does it need to be signed by the party to be charged? If you are
trying to show proof of a K, how can you show mutual assent unless
the evidence you are presenting is signed by the other party?
(ii) Merchant’s Exception in UCC 2-201 (2): if there is an agreement
between two merchants and one sends to the other a confirmation of a
K, a failure to object to that confirmation makes the K binding on both
parties.
4) If so, does the writing indicate the essential terms of the K? UCC 2-201
requires only quantity)

E) What Kind of Writing is Required?


1) A written agreement OR;
2) A written memorandum of the agreement: maybe there were letters or memos
exchanged which show proof of an agreement (however, the memo or letter
must be signed by the party to be charged)
3) Doesn’t necessarily need to be a written K.
4) Generally, the writing must contain the essential terms of the agreement it
memorializes.
5) The writing doesn’t always have to be in one document, you can piece it
together with several different documents, although at least one of the
documents needs to signed by the party being charged.

F) UCC 2-201: the UCC also requires a writing for a sale of goods for the price of
$500 or more. BUT:
1) It is more lenient: instead of requiring the essential terms, it only requires that
the quantity of goods is specified (the specific price is NOT required)
2) UCC was conceived with merchants in mind, even though it applies to all
sales of goods over $500. It is more lenient and ‘murky’ because business has
to flow, we can’t have merchants checking with their lawyers over every
single term when they are making transactions.
3) Merchants exception: between merchants, it can make a “writing” efficacious
against a non-signer. If a written notice of confirmation is sent from one
merchant to another, and he makes no objection, the agreement is binding
even without his signature on the confirmation.

G) Ameliorating the Operation of the Statute


1) Monarco v. Lo Greco: Monarco was “estopped” from invoking the Statute
of Frauds in order to defeat a definite, mutually assented to oral
agreement between Natale and Christie because Christie had given 20
years of labor in reliance upon Natale’s promise to give him his half
interest in the land.
(i) This oral agreement required a writing because promises made after
death require a writing (it should be in a will).
(ii) Normally, we would say “tough luck”; Christie should have gotten the
promise in writing; but the Court “ameliorates” the Statute here.
(iii) They use promissory estoppel more as an analogy rather than in
practice: there is C for this promise, we are not looking for an alternate
basis of enforcement; if PE is strong enough as an alternative basis for
enforcement, it should be strong enough in a situation where there is
C, definiteness, and mutual assent.
(iv)Christie clearly relied to his detriment (he could have gone to college
or bought his own land), and Natale was unjustly enriched (he got all
the benefit of the labor and then left Christie out to dry).
(v) Rule: the Statute of Frauds can be estopped where the Pe was
induced to seriously change his position in reliance upon the
promise, and where the Pr is unjustly enriched.
(vi)Statute of Frauds matters, it is a concrete rule, but it will give in
extreme circumstances in order to supply fairness and justice.
2) Halstead v. Murray: the court holds that an attorney and his client have
such a special “unity” relationship that an the attorney’s initiation of a
settlement upon oral approval from his client is sufficient to make the
settlement enforceable even though the client did not sign off on it.

III) POLICING THE BARGAIN


A) Policing the bargain could also be called: “Bases for NOT Enforcing a Promise.”
B) Bases for NOT Enforcing a Promise:
1) Status: persons of certain status may not be bound by the agreements they
make.
(i) Capacity (e.g. age, mental illness)
(ii) Relative bargaining power: someone with a clear disadvantage of
relative bargaining power may not be bound
2) Behavior: certain behavior of the other party may be a cause for voiding K.
(i) e.g. duress or misrepresentation
3) Substance: if the agreement seems clearly unfair in substance, such that no
reasonable person would have entered the agreement, the K may be
unenforceable.
4) You won’t see many cases where just one of these factors is applied; usually 2
or 3 will be applied in order to show conclusively that a K is not enforceable;
if you hit all three, you have a pretty good case that K is not enforceable.
C) Policy Reasons for “Policing” the Bargain: there must be more of a rationale
than just: “that bothers me, so we should do something about it.” Arguments for
“fairness” must be made on sophisticated legal grounds.
1) Rights: we have rights guaranteeing autonomy and “free will”; we should
have free will to make a K
(i) One could argue that this rationale should mean that policing should
be limited in order to allow for total autonomy
(ii) Others could argue that certain persons, such as minors, are not mature
or knowledgeable enough to exercise that free will properly.
2) Doctrine: to make sure that the doctrinal requirements of the bargain are met;
e.g. mutual assent, consideration, etc.
(i) One could argue that a mentally ill person is not cognizant enough to
be said to “assent” to a bargain; thus there can be no mutual assent.
3) Economic Efficiency: ensuring that bargains made are efficient and beneficial
to both parties
4) Legitimacy of Law:
(i) Fairness
5) Institutional Competence: what is the proper institution and institutional rules
for policing the bargaining process?
(i) Courts v. Legislatures: is it the proper place of the Legislature or the
Courts to determine where the lines should be drawn?
(ii) Rules v. Standards: should we draw bright-line rules for governing the
bargain or should we examine each case to see if the facts fit with a
broader standard?
D) Capacity
1) Minors; Kiefer v. Fred Howe Motors: 20 year old’s agreement to buy a car
was voidable and unenforceable; he was allowed to get out of the
agreement and get his money back when the car had problems, even
though he was only days away from his 21st birthday when he made the
agreement.
(i) Modern approach is that K’s made by persons w/o capacity are
voidable: they can be backed out of at any time; there is a K as long as
the person w/o capacity confirms that K
(ii) Court recognizes that it may be prudent to re-analyze the rule that says
anyone under 21 is a minor and has no capacity to form a K, but it is
the Legislature’s place to make such a change.
(iii) Sometimes we want to be more subjective. But, subjectivity breaks
down the rule, and starts a slippery slope such that the rule ceases to be
effective. We want flexibility but a line has to be drawn somewhere.
2) Mentally ill; Ortelere v. Teachers’ Retirement Bd.: woman’s decision to
change her retirement plan, so that her monthly payment would increase,
but her husband would receive nothing upon her death, was not
enforceable because she was suffering from a mental illness at the time.
(i) Modern approach says that if you understand the process, but are
unable to make a reasonable, rational decision about the K because of
mental illness, AND the other party has knowledge of your condition,
K can be voidable.
(ii) A person would not have done what Ortelere did if they were acting
reasonably; it doesn’t make sense
(iii) Dissent argues that because Ortelere had sent a detailed letter
asking questions about the plan it seems that she made an informed
and rational decision; she knew what she was doing.
3) Mental illness rule: K duties are voidable by a party if, due to mental illness,
that party was unable to understand the transaction OR understood but was
unable to act in a reasonable manner in relation to the K and the other party
has reason to know. Restatement 2d Sec. 15.
(i) Cundick v. Broadbent: the Court held that Cundick’s K to sell his
land to Broadbent for a low price was enforceable even though
Cundick had a mental condition.
1. What is the difference between this case and Ortelere? In both
cases the “incompetent” person showed some knowledge and
savvy in the transaction, yet the court finds Ortelere void and
Cundick’s K enforceable.
2. One distinction between this case and the Ortelere case is that
Ortelere’s mental condition was well known to her employers,
whereas Cundick had tried to hide his mental condition.
3. Another distinction is that in Ortelere, the wife made the
decision behind her husband’s back, but here the decision was
made in front of Cundick’s wife and a lawyer.
4. The dissent argues that the price was so unreasonably low that
Broadbent should have realized that Cundick was not “all
there.”
5. Perhaps the better is distinction is one of fairness: How far “up
the creek” are the parties because of the transaction? In
Ortelere, the husband is put in a pretty bad position as a result
of his wife’s decision; whereas it is not as bad in Cundick (at
least he got some money).
E) Overreaching: when you already have a deal, and one of the parties tries to
change that deal at the last minute, and the other party agrees. The question is: Is
that agreement the consequence of overreaching or was it voluntary?
1) How do you determine whether or not the agreement was overreaching or
voluntary? Two approaches:
(i) Procedural: how did they go about changing it?
(ii) Substantive: does the substance of the change seem fair, or does it
seem to undermine the other parties’ interest?
2) Pressure in Bargaining
(a) Pre-Existing Duty Rule: there is no consideration for a promise to fulfill a
duty which is already owed; a mechanical application of the doctrine of
consideration.
(i) Ex. Alaska Packers v. Domenico: workers had agreed to work on a
salmon boat for $50; when they got to Alaska they demanded $100 or
they would refuse to work; unauthorized agent agreed to the increase
bec. they had no time to find other workers; Alaska later refused to
pay the increase; Court held that the agreement for a price
increase was unenforceable because the workers had offered
nothing more in consideration to receive the increase.
1. There was clearly a coercive environment here; workers knew
that Alaska Packers needed them, they squeezed an increase
out of them.
2. Court applies pre-existing duty rule in order to counter this
coercive practice.
(ii) This strict rule can be modified under certain circumstances;
Schwartzreich v. Bauman-Basch, Inc.: Schwartzreich had contracted
for a one-year salary of $90 a week; Before he began performance, he
was offered a similar job for $115 a week; He told Bauman and they
agreed to pay him $100 a week if he would reject their offer; they then
prepared and signed a new contract for the same performance but for
an increase in pay. The court upheld the new contract for $100 a
week even though it was a contract for the same amount and type
of work that was contracted for in the first agreement.
1. The facts don’t point to a coercive environment; Bauman
voluntarily agreed to the new terms and even went through the
trouble of ripping the signatures off the old K and forming a
new K.
2. This holding violates the pre-existing duty rule, but you can see
under the facts why this might be distinguished from a case
like Alaska Packers.
3. This is an example of rescission and modification.
(b) Rescission & Modification: when an existing K is terminated by consent
of both parties and a new one is executed in its place upon the same
subject matter, the new K creates legal obligations. The problem is: when
can we say that there was a rescission and modification and when should
we apply the pre-existing duty rule?
(i) Ex. Watkins & Son v. Carrig: Watkins and Carrig agreed to a certain
price for Watkins to excavate under his house; Watkins encountered
solid rock; Carrig orally agreed to pay a higher price for excavating
rock; Carrig later reneged on the price increase; the Court enforces
Carrig’s promise to pay a higher price for Watkins to excavate a
cellar under his house, even though Watkins already had a pre-
existing duty to do so.
1. One way of looking at this case is that Watkins in fact had to
do more work than was understood under the first K. No one
anticipated solid rock under the house.
2. Carrig was not forced to pay more money, they came to a
mutual agreement that more money should be paid to take care
of the solid rock.
3. The point of the pre-existing duty rule is to prevent coercion, it
should not be used where there is no coercion present.
4. Watkins didn’t get anything that was unfair, he was paid fair
market value for excavating solid rock.
5. This conclusion is fair in light of the circumstances: the fact
that he did have to do more work that previously anticipated
(ii) When is there a proper rescission and modification?
1. Williston: there must be a real gain to both sides; modification
is not enforceable unless both sides got something
2. Schwartzreich: as long as there is a formal termination of the
previous K; e.g. “tearing signatures off the previous K.”
3. Watkins: any voluntary change in K is okay; look at
circumstances to see if there was a voluntary, consented to
change in the K. This view is adopted by Restatement.
4. Another rule (Professor): as long as there is some change in
consideration. Ex. if one agrees to do the work in a different
way than previously agreed. B owes P $100, P says “Give me
$75 and do something that I want.” There may be no net
benefit to P, but there is a change in consideration.
(c) Duress; Ex. Austin Instrument v. Loral Corp.: the Court rejected the
agreement for a price increase between Loral and Austin in which
Austin threatened to stop delivery of essential gears if Loral did not
agree to pay a price increase; they held that the agreement was not
valid because it was made via economic duress.
(i) The pre-existing duty rule does not apply because Loral has already
paid the price increase and they are trying to get that money back; they
didn’t promise to pay the increase (like Alaska Packers), they actually
did pay the increase.
(ii) Loral supplies equipment to the armed forces; they may have lost their
biggest customer, the Navy, if Austin had followed through on its
threat not to deliver the precision gear components.
(iii) Moreover, Loral could not find any other subcontractor that would
be able to deliver the gear components on time
(iv)As such, the agreement was void through the application of duress.
(v) Economic Duress:
1. Party is forced to agree by substantial threat
2. Threatened party must be put in a situation where they would
not be able to obtain the goods elsewhere.
(d) Undue Influence; Ex. Odorizzi v. Bloomfield School District: Odorizzi’s
agreement to resign as a teacher after being arrested for homosexual
activity was void because it was made under an environment of undue
influence.
(i) Elements of Undue Influence:
1. High pressure situation
2. Consenting party has some “weakness”
3. Other party takes advantage of that weakness
4. There is an unfair advantage of one party over the other.
(ii) Characteristics which help to show Undue Influence:
1. Discussion of transaction at an unusual time
2. Consummation of the transaction in an unusual place
3. Insistent demand that the business be finished at once
4. Extreme emphasis on untoward consequences of delay
5. Use of multiple persuaders
6. Absence of third-party advisers
7. Statements that there is no time to consult attorneys or financial
advisers.
(iii) Many of the elements and characteristics were present in the
Odorizzi case: Odorizzi had been up for 40 hours and right as he gets
home he is confronted by his school board members asking him to
resign, telling him there is no time, that if he refuses they will fire him
and publicize his arrest.
3) Concealment and Misrepresentation
(a) Generally, a seller is not obligated to disclose all details of the sale that
may be detrimental to the buyer or vice versa.
(i) Traditional common law rule is caveat emptor: let the buyer beware.
(ii) Swinton v. Whitinsville Sav. Bank: agreement to buy a house was
not voided by the fact that the house was infested by termites and
the seller knew of that fact; court holds that the seller is not
required to disclose the termite problem.
1. The key fact in this case is that the buyer did not ask
2. The buyer did not misrepresent either; he simply concealed the
detrimental fact
3. If the seller had said, “I’ve lived here for 50 years and there has
never been a termite problem,” that would be a clear
misrepresentation and the K would probably be rescinded.
4. This rule is not followed in every state. Many states have
modified this rule to one extent or another.
(iii) Latent defects; some states (including CA) require disclosure of
latent defects: defects which are not readily visible to the buyer.
1. The policy reason behind this is one of preventing economic
waste: if the seller discounts the price a little bit because of the
defect, lets the buyer know, the buyer can fix the problem
immediately instead of waiting until the problem exacerbates
further and ruins the economic viability of the house.
2. Problem: how do you define “latent defect”?
(iv)Kannavos v. Annino: Annino (D) advertised her property as an
investment property with several apartments that could be rented out;
Kannavos bought the property; City notified Kannavos that the zoning
would not allow the property to be used as apartments; court held
that the agreement to buy the property was void because Annino
had misrepresented the character of the property.
1. How is this case distinguishable from the Swinton case?
2. Swinton only concealed the defect; Annino actually tried to sell
her property as an investment property knowing full well that it
was illegal to use the property in that way.
3. The so called defect in this case is something that the seller led
the buyer to believe was not a defect at all.
4. The seller counters that it was public record what the zoning
was; however, by telling the buyer that the property is great for
renting out apartments and that it has been used in the past for
this purpose, there was no reason really for the buyer to check
out the zoning ordinance.
F) Adhesion Contracts and Unconscionability
1) Issues with Consumer Contracts
(i) You contract for so many things in your life: rent, phone, gas and
electric, buy a car, etc. it goes on and on.
(ii) It used to be that a contract was when farmer John and farmer Bill met
in a field and negotiated a deal; but now you contract almost all the
time with people that you have never even met. Most of the time you
don’t even have the opportunity to bargain.
(iii) Contrast between Step Saver and ProCD case: one crucial way in
which they were different was that Step Saver involved two businesses
that had a relationship and ProCD involved a dispute between a big
company and one individual guy.
1. ProCD was a consumer contract; there was no negotiation
between the company and the guy who bought their program,
there was just a warranty and terms inside the box and the guy
was said to be bound by them once he opened the box.
2) Problems with Adhesion Contracts; O’Callaghan v. Waller & Beckwith
Realty Co.: Court refused to nullify a standardized K, agreed upon during
a housing shortage, between landlord and tenant which contained an
exculpatory clause which exempted the landlord from any liability for
injury caused to tenants as a result of landlord’s negligence.
(i) O’Callaghan makes the argument that this exculpatory clause is
against public policy and thus that portion of the K should be void.
(ii) Court says it is up to the legislature to establish a determination of
public policy in regard to exculpatory clauses, not the court.
(iii) Though there is a housing shortage at this time, housing
availability moves in cycles and thus it is not a permanent state of
affairs.
(iv)O’Callaghan should be held to the K because she never even addressed
the clause and never stated any problem with it when she first
contracted to rent the apartment.
(v) Public Policy: is it really a good public policy to allow landlords to not
be responsible for the upkeep of their grounds, such that if someone
gets hurt because of their negligence they cannot be held liable?
(vi)Dissent: housing shortage give landlords unfair bargaining advantage;
almost all landlords in the area were using these clauses; O’Callaghan
was left with little choice in accepting the K.
3) Distinguish O’Callaghan with Klar: no one really reads a coat check stub, but
people do or at least they should read a contract for a lease.
(i) An exculpatory clause is not something that you would see on a coat
check stub; but it is reasonably something that may come up in a lease
contract for an apartment.
(ii) You should expect to find a lot of important things in a lease, so the
argument that most people don’t read the whole lease does not really
hold as much water because you should read the lease and know its
terms.
4) Another argument is that the reason we can rent apartments cheaper, or offer
coat check service cheaper or computers cheaper is because we don’t
negotiate over every single detail involved in the transaction; if we did, it
would probably mean that prices would go up (because contractors would
have to pay to make sure that all of their customers were fully aware of the
terms).
5) Distinguish Adhesion K’s from “unconscionability.”
(i) Adhesion K’s are simply standardized forms imposed by a stronger
bargaining party upon a weaker party, such that there is a perceived
lack of choice for the weaker party.
(ii) many K’s are adhesion K’s; most are enforceable
(iii) unconscionable is a legal term for unfair
(iv)only some adhesion K’s are found to be unconscionable and thus
unenforceable
6) Graham v. Scissor-Tail, Inc.: the Court held that the K between Scissor-
Tail and Graham was an adhesion K; the arbitration clause in the K was
something that Graham could “reasonably expect” but it was
unenforceable because it was “unconscionable.”
(i) It was an adhesion K: 1) Graham could not hire the musician unless he
signed the form; 2) the union controlled most artists, so he could not
go anywhere else without being faced with the same form.
(ii) Adhesion K’s can be unenforceable if 1) the provision does not fall
into the “reasonable expectation” of the weaker or “adhering” party;
OR 2) the K is oppressive or “unconscionable.”
(iii) “Reasonable expectation”: Graham had been a party to thousands
of AFM contracts and had appeared before the AFM board before.
(iv)“Unconscionable”: the arbitration clause was unfair because it required
disputes to be settled by the union’s own board
7) Henningsen v. Bloomfield Motors, Inc.: the court found a dealer’s
disclaimer of implied warranty unenforceable because it was
unconscionable.
(i) This was a true adhesion K: P had no choice but to sign the form when
he bought the car; all automobile manufacturers were part of the AMA
and they all had similar clauses in their sales agreements
(ii) Status: the big manufacturers had gotten together and created this
crafty, uniform warranty that would protect them.
(iii) Note that this kind of collusion was not present in the O’Callaghan
case.
(iv)Behavior: the disclaimer was in six point type and it was written in
such a way as it was very difficult to understand. It almost sounded as
if it was giving you a warranty on parts, but it was actually limiting
liability only to replacement of defective parts, period.
8) Hill v. Gateway: Court upheld a Gateway arbitration clause, included in a
warranty agreement that was inside a shipping box along with the
computer itself; the Court made a ProCD analogy in making their
decision.
(i) The Court did not even do an unconscionability analysis; they
examined the case using a mutual assent analysis
(ii) This is an example of a court that looks more at procedure rather than
substance; they look to see if there was anything wrong in the
procedures that led up to the agreement and ignore the substance of the
agreement itself.
(iii) This was an adhesion K, but the court makes no mention of the fact
in their decision.
(iv)In a related case, Brower v. Gateway, the court did more of an
adhesion/unconscionability analysis and they found that the substance
of the arbitration was unconscionable.
(v) The arbitration in dispute was unreasonable: the arbitration forum was
in Chicago, but the International Chamber of Commerce is based in
France. The rules of the ICC are available but only if you put up
$4,000, $2,000 of which is non-refundable.
(vi)Professor doesn’t like the analogy of this case to ProCD: it is easier to
take back a software program, but to ship a computer back to Gateway
can be expensive and it is much more burdensome.
(vii) Easterbrook contends that the Hill’s should have known about the
warranty through advertisements OR they could have called the
vendor and asked them to send a copy of the warranty.
(viii) He says that the Magnuson-Moss Warranty Act requires firms to
distribute their warranty terms upon request.
(ix)However, the Magnuson-Moss Act also requires that the warranty be
fully publicized in ads and outside the store if there is an arbitration
provision; Gateway had failed to do so.
G) Unconscionability
1) Views of Unconscionability
(i) Traditional view: some schools of thought believe that
unconscionability should only apply to procedural matters, and that the
substance of the K cannot be looked at in using an unconscionability
doctrine. This is the view that Easterbrook follows.
(ii) Modern view: The view that professor has shown us says that there is
a mixture of procedure and substance in looking at whether or not a K
should be ruled unenforceable in accordance with unconscionability.
(iii) Hands-off approach: There is also the view that it should be left
up to the legislature to designate what kinds of provisions are
unconscionable, but as you can see with the Magnuson-Moss Act
below, often people don’t have a full understanding of the statute.
2) Efficiency v. Ideology
(i) Efficiency: is it more efficient to let the free market govern contracts
OR to interfere where justice requires using unconscionability?
(ii) Ideology: “freedom to contract is an important value, not only because
it is efficient but because it is morally good” / “fairness is more
important than freedom to contract and the government must step in to
ensure fairness of contracts.”
3) Private v. Public
(i) In an unconscionability analysis there is conflict between the law,
which is within the sphere of the government (public), and contract,
which is perceived as being in the sphere of private institutions.
(ii) However, contracts are only enforceable by the law, so they are not
wholly private.
(iii) Furthermore, there are situations in which the public interest may
need to trump the individual interests of a private agreement. Ex.
agreement to sell drugs, etc.
(iv)A rules-based system is predictable, but too rigid; a system that takes
into account “fairness” is less predictable but allows flexibility.
4) Williams v. Walker-Thomas Furniture Co.: the court rejected a pro rata
credit scheme adopted by the Walker-Thomas Furniture Co. as
unconscionable.
(i) Consumers, mostly poor people, were allowed to buy multiple items
on credit, but if they defaulted on any payment for the balance of all
items, the store could repossess all of the items regardless of the fact
that the value of some of the items had already been paid.
(ii) Most of the persons using this system were on government welfare
programs; the store would come around once every month to collect
on a portion of those welfare checks.
(iii) Court looked at the bargaining power between each party as well
as the manner of formation in deciding that the clause was
unconscionable
(iv)The language of the pro rata clause was vague and confusing and
maybe the consumer did not have full knowledge of the terms
(v) Dissent: majority is assuming that the poor consumer is stupid; it
furthers a stereotype that may cause the poor to have less access to
credit.
(vi)Prof Note: lenders actually like to lend to people with bad or no credit
because they are justified in charging higher rates and people are more
likely to default at those higher rates.
H) Public Policy
1) This involves K’s where two parties agree to a contract, but the enforcement
of that agreement would undermine the interest of the public, so the court
voids the agreement.
2) Bovard v. American Horse: the court voided an agreement between P and
D to buy a store which sold pipes and bongs that could be used to smoke
marijuana on the grounds that the agreement undermined a state statute
prohibiting the use of marijuana.
(i) This is a bit of a stretch because there is no law specifically outlawing
pipes and bongs, only the smoking of marijuana. They reason that the
sale of drug paraphernalia may encourage and further marijuana use.
(ii) The ironic thing about this is that one of the parties is trying to get out
of the agreement, and he is trying to do so in the name of “society.”
(iii) There is also this bigger picture of “fairness” involved in the
decision because P has received back much of his machinery and most
of his jewelry business, so it’s not like he has been hung out to dry.
3) Hopper v. All Pet Animal Clinic: Hopper left the veterinary clinic and started
her own despite a 3 year non-competition covenant she had signed earlier;
the court upheld the enforceability of the non-competition covenant, but
reduced its length from 3 years to 1 year.
(i) The idea is that a non-competition clause is bad for society because it
reduces competition in the marketplace and helps create a monopoly.
(ii) Dr. Hopper’s argues that the clause cannot be enforced because 1) she
has a right to make a living, freedom to work and 2) this clause also
hurts the public because it prevents her from providing her expert
services to the general public.
(iii) Court reasons that a non-competition clause is reasonable if:
1. it no greater than is required for the protection of the employer;
2. does not impose undue hardship on the employee
3. it is not injurious to the public
(iv)The court finds that the 3 year durational period is unreasonable, but
instead of rejecting the clause, they hold that the non-competition
clause is enforceable, but only for a 1 year period instead of 3 years.
(v) The court essentially rewrites the contract; this is called reformation;
they are basically saying that the contract should have been written
this way and that the two parties did it wrong.
(vi)Employers might actually like this holding because it allows them to
push the envelope; most employees are not going to bring a suit, and if
they do the court may just strike out a portion of the K, but still
enforce it.
4) Simeone v. Simeone: wife is trying to void a prenuptial agreement on the
grounds that it is unconscionable and against public policy; the Court
rejects an older conception of prenuptials and holds that a prenuptial
agreement must be treated as a K; they uphold the enforceability of the
prenuptial agreement.
(i) The court disregards the old test which said that the prenuptial
agreement is valid if either 1) it made a reasonable provision for the
spouse OR 2) was entered into after a full and fair disclosure of the
general financial positions of the parties and the statutory rights being
relinquished.
(ii) The old rule was based on a view that women needed to be protected
because they were less knowledgeable about contractual matters.
Modern public policy demands that women be treated equally and be
held equally responsible for contracts they enter into.
(iii) The court seems to take a much more procedural view of
unconscionability: a K can be rescinded but only if there was fraud,
misrepresentation or duress, if there was something wrong with the
formation of the K; it cannot be voided simply because of the
substance or “reasonableness” of the bargain.
(iv)The court determined that the wife knew full well what she was getting
into when she signed the prenuptial agreement as evidenced by the fact
that it had been discussed several times before the wedding day.

IV)REMEDIES FOR BREACH


A) Measuring Expectation Damages
1) Two Formulas:
(i) Formula A: Damages = loss in value + other loss – cost avoided – loss
avoided
(ii) Formula B: Damages = expected profit + other loss + cost of reliance –
cost avoided
2) Definitions
(i) Loss in value: What is the difference between what P should have
received and what P actually got?
(ii) Other loss: there may be other losses caused by the breach of the K;
(iii) Cost avoided:
(iv)Loss avoided:
3) Ex. Sullivan v. O’Connor; Doctor fee: $300 (one time); Hospital Fee: $100 (3
ops.); Physical Pain: $3,000 per op (3 ops.); Promised improvement in
appearance: $20,000; Loss in appearance due to doctor: $10,000
(i) Formula A is easier to use: $20,000 + ($10,000 + $100 + $3,000) =
$33,100 in expectation damages
(ii) Loss in value is whatever the value of a correctly performed surgery is
(in this case $20,000), subtract what she got, which was basically $0
because it actually made her look worse.
(iii) There were also other losses: her looks worsened ($10,000), and
she had to endure three operations instead of the promised two ($100
hospital fee for third operation and $3,000 pain and suffering for third
operation).
(iv)There were no costs avoided or loss avoided because she went through
the entire operation; she did not stop halfway through and thus avoid
certain costs or losses.
4) Expectation Damages in the UCC
(i) Buyer Remedies: See Laredo, UCC 2-712, 2-713
(ii) Seller Remedies: See Diasonics, UCC 2-708, 2-718
5) Buyer Remedies Hypo: Seller Breaches K for Sale of Goods
(i) On Jan. 1, Joo make a K to sell 20 bikes to Robinson for a total of
$5000 (the market price at the time of K). Delivery on Feb. 5, pymt on
delivery. On Feb. 5, J informs R that he will not deliver, mkt. price has
gone up to $5600.
(ii) What is R entitled to under UCC? UCC 2-713: Buyer’s remedy if
seller breaches: “difference between mkt. price at the time buyer
learned of the breach and the contract price, together with any
incidental and consequential damages provided in this article, but less
expenses saved in consequence of the seller’s breach.”
(iii) Thus, under UCC 2-713, assuming no incidental damages or
expenses saved, R would be entitled to $600.
(iv)R expected to get bikes on Feb. 5 (he made a gamble). He didn’t get
those bikes and now he has to go and buy them from somewhere else
where they will cost $5600. So, in order to protect his expectation he is
entitled to the increase in market value of the bikes.
(v) Also, R scored: without knowing that the market value would increase,
he fixed a price at $5000; when they increased by $600 he collected on
his gamble and he should be entitled to that extra money.
6) Another Hypo: Let’s say R goes out and buys substitute bikes for $5700 with
a $100 rush fee.
(i) UCC 2-713: Feb. 5 mkt price – K price = $5600-$5000 = $600
(ii) UCC 2-712: “Cover” remedy: $5700-$5000+$100 = $800.
(iii) Under 2-712, R would get more in damages that he would get
under normal circumstances; as long as R went out immediately and
made a reasonable effort to get a good price on the bikes, he can use 2-
712 in calculating his damages.
(iv)Does it make sense to have two different formulas? The difference is
really one between theory and practice. 2-713 is totally consistent with
the theory of what R would lose because of the breach. 2-712 is a more
practical formula, because in most cases the buyer will have to get the
product anyway, so it helps to address this practical reality.
7) Seller Remedies Hypo #1: Buyer Breaches K for Sale of Goods
(i) Robinson has one bicycle that cost him $275. He contracts to sell it to
Joo for $300. J breaks the K by refusing to buy.
(ii) R’s damages? R is entitled to $25 because he expected to get $300, but
he also expected to lose the bike which is worth $275 (in other words
he had $275 of costs avoided). The $25 is also referred to as expected
profit.
(iii) UCC 2-708 (1): seller is entitled to the difference in the contract
price less the market price.
(iv)If this is not adequate to compensate, UCC 2-708 (2) allows seller to
collect the profit (including reasonable overhead) which the seller
would have made from full performance by the buyer.
(v) Let’s say that R then sells the bike to X for the same price, $300. What
are R’s damages now? He is not entitled to the $25 lost profit because
he already got that profit and he only has 1 bicycle anyway. He may be
entitled to any incidental damages that he incurred in finding another
buyer (storage costs, advertisements, etc.).
(vi)What if R sells the bike for less than $300? R is entitled to the
difference between the K price and the price he sold it for in addition
to incidental damages (UCC 2-706).
8) Seller’s Remedies Hypo #2
(i) R’s bike factory makes bicycles at a cost of $275 each and sells them
for $300. The factory can make an unlimited number of bikes at this
cost. I contract to buy a bike for R for $300. I break the K by refusing
to buy. X walks in and buys a bike for $300
(ii) R sues me. What are his damages? He is entitled to the expected profit
because I was going to buy one of many bikes and X was going to buy
one of many bikes, but because I breached only 1 bike got sold instead
of two.
9) Seller’s Remedies: Hypo #3
(i) If R’s factory is making bikes such that the cost to make additional
bikes would not be giving him any more profit, then such a breach
would not entitle R to damages including expected profit because he is
basically back in the same situation as he was in Hypo #1; it is as if he
only has 1 bike instead of unlimited bikes.
(ii) This was the situation being discussed in the Diasonics case.
10) Losing Contracts Hypo: Owner promised to pay Builder $1mil; B promised to
build building; when B has spent $500K on construction, but not yet
completed, O breaches; O has paid B nothing. It would have cost B another
$600K to complete the building.
(i) What remedy should B get?
1. Formula A: Lost value = $1 mil, cost avoided is $600K, total
cost would have been $1.1mil
2. Formula B: Expected profit is -$100K, currently at -$500K
(cost of reliance/work done already)
i. B should get $400K expectation damages
ii. Burden of proof is on O to discover the expected profit
(ii) O gets $500K of construction work for only $400K. Is it fair?
1. Policy – avoid unjust enrichment, give $500K to B
2. Has O really gotten $500K worth of construction? $500K is
money spent, but not necessarily the reasonable value of the
services.
B) Limitations on Damages
1) Avoidability: an injured party cannot just blatantly run up the damages after
the other party breaches, when it is possible to avoid those damages.
(a) Rockingham County v. Luten Bridge Co.: County contracted w/ Luten to
build a bridge; as construction was in progress, County backed out; Luten
completed the full bridge anyway; Luten is not entitled to the full
contract price or the full value of the bridge because they could have
avoided those costs by stopping work immediately and suing for
breach of K.
(i) In order to satisfy Luten’s expectation, they need to get the profits
expected from the completion of the bridge and they should recover
the value of the work before County’s breach. But, they should not be
able to recover the total cost of the bridge because they could have and
should have avoided those costs.
(ii) Building a bridge is a service not a good under the UCC. Goods are
treated differently. If a buyer repudiates a contract for sale of goods
and the seller has already begun manufacture of the good, the seller
can finish the manufacture “in the exercise of reasonable commercial
judgment for the purposes of avoiding loss and of effective
realization.” Seller could then look for another buyer and if he found
one he could recover the difference between the price sold and the
original contract price.
(b) Tongish v. Thomas: Tongish agreed to sell seeds to Coop for a total of
$9,000. C was contractually obligated bound to resell to Bambino. Mrkt.
price went up to $18,000. T breached K and sold elsewhere for $14,000;
the court holds that Coop is entitled to the larger remedy under UCC
2-713 ($9,000) rather than simply expected profit from the K ($455).
(i) Two possible remedies for Coop:
1. Expected profit on Bambino K = $455
2. Mrkt price – K price ($18,000-$9,000) = $9,000.
(ii) Previous case, Allied Canners, held that the remedy for this kind of
situation is “expected profit” not “difference between market value
and contract price.”
(iii) The court distinguishes Allied Canners. There, the third party
buyer had agreed to rescind their K; whereas in Tongish, Coop was
still obligated to provide seeds to Bambino. The only way they can get
the seeds is to pay market price for them or else they will be sued by
Bambino for the market price. Thus, they are not really being
compensated very well if they only get $455.
(iv)Public Policy concerns: court wants to encourage parties to honor their
contracts; encourages market stability.
(v) Finally, if there is going to be a windfall to a party it should go to the
party who kept to the bargain and not to the bad faith party.
(c) Parker v. Twentieth Century-Fox Film Corp.: Shirley McClaine signed a
K with Fox to star in a song and dance movie; Fox breached and offered
her another movie deal involving a western; McClaine denied the new
offer and sued for breach of K; the Court held that McClaine was not
obligated to accept the new offer because the employment was
different in kind and inferior to the originally contracted
employment; she was entitled to recover the difference between the K
salary and what she actually earned or reasonably could have earned.
(i) In order to show that damages could have been avoided, the employer
must show that the alternate employment was comparable, or
substantially similar, to that of which the employee has been deprived.
The court further adds that in order to be similar the other employment
cannot be different in kind or inferior employment.
(ii) The court reasoned that the other offer of employment was different in
kind because the talents and skills in a song and dance movie are
different from that required in a western movie. It is inferior in that she
had more control over the director and scripts in the first movie and
she does not have those same powers in the second offer.
(iii) Dissent: it is not that obvious that the work is inferior or
different… it may make it different or inferior but we need to have an
actual trial in order to determine that fact; we should not just grant
summary judgment to the plaintiff.
(iv)Another key difference between the employment opportunities was
that the original movie would allow her to stay in CA and the second
movie forced her to travel to Australia.
(v) Another key fact is that she had already “expressed an interest in
performing the role in the Big Country movie” before she agreed to the
song and dance movie.
(vi)Is it significant that it is the same producers who breached the first
contract who are offering the second contract? Neither opinion deals
with this fact. The question is whether they planned to do this the
whole time. Did they really just want her to do the western and set her
up by offering the song and dance movie first?
(d) Jacob & Youngs v. Kent: Kent (D) contracted w/ J&Y (P) to build his
house; Kent stipulated that all pipes should be Reading steel pipe; P
failed to do so; D discovered the defect and refused to make a final
payment on the house; P sued; the Court held that p was entitled to the
remainder of his money; but D is only entitled to the difference in
value between a house with Reading pipes and a house without
Reading pipes; he is not entitled to the cost of repairing the defect.
(i) The difference in value would be relatively small, but the cost of repair
would be staggering because it would require knocking down walls in
the house, replacing the pipe and putting walls back up.
(ii) Three factors:
1. Usually they will award the cost of the repairing the defect, but
when that cost is grossly disproportionate to the actual loss in
value they will award only the difference in market value.
2. If the imperfection is “trivial” they will also be more inclined
to use the loss in market value.
3. Finally, they will use the market value formula when the defect
was not done intentionally.
(e) Groves v. John Wunder Co.: Groves allowed Wunder to extract gravel
from his land, but the K provided that Wunder would refill and regrade
the land after he was done; Wunder failed to do so; the court awarded
Groves a remedy equal to the cost of repairing the defect in the land
($60,000) even though the loss in value to the land was minimal
($12,160).
(i) The issue was raised differently here than it was in Jacobs & Young.
There the issue was raised as a defense for not paying the contractor,
here it is raised in the complaint.
(ii) One way to look at damages is loss in value subtract cost avoided (LV
– CA). That is not easy to do here.
(iii) Why does the court decide here that the proper damages are the
cost of fixing the defect?
1. There was a willful breach, it was not an accident.
2. They breached a fundamental purpose of the K.
3. Value to owner may not be market value, there may be
sentimental value.
4. The issue is not the value of the land, the issue is that Wunder
failed to fulfill his end of the bargain.
(iv)One problem with this view is that owners of land may do things to
their land that are not necessarily beneficial to the overall value of
their property, but they want them done for other reasons. Ex. Bob
Villa and the swimming pool in a quaint Connecticut bungalow: “my
daughter loves to swim.”
(v) We are not completely sure why Groves entered into this contract?
Was the sole purpose to profit off of Wunder’s extraction of gravel or
was the cleaning up of the land a key component of the contract; was
that term bargained for?
(vi)Maybe it would be better for the court to order specific performance
because that would encourage the two parties to negotiate a settlement
between them which would be somewhere in between the loss in
market value and the cost of repair.
(f) Peevyhouse v. Garland Coal & Mining Co.: Peevyhouse entered into a K
with Garland allowing them to strip mine coal on their property; K said
that G must restore land upon completion; G failed to do so; cost of
repair was disproportionately higher than loss in value; court held that
Peevyhouse was only entitled to damages equal to the loss in value of
their land.
(i) The facts are very similar to the Groves case, but the outcome was
entirely different.
(ii) Like the Jacobs and Young case the court notes the “grossly
disproportionate” difference between the loss in market value and the
cost of repair.
(iii) Was restoring the land “incidental” or “fundamental” to the
completion of the contract? The plaintiffs had specifically asked that
the coal company restore the land. In fact, they said that if the coal
company did not agree to the term they would not sign the contract.
(iv)Yet the court seems to believe that the fundamental purpose of the K
was to make $ off of the extraction of coal; they focus on what they
believe to be the purpose and intent of the parties.
(v) Interesting fact: the Peevyhouse’s actually gave up money in order to
get the company to agree to a clean-up. Peevyhouse’s lawyer must
have failed to this up – bad lawyering.
(vi)The subjective connections to the land seem to be more applicable to
the Peevyhouse case than to the Groves case.
2) Foreseeability
(a) Hadley v. Baxendale: a mill’s crankshaft was broken; they contracted
with a delivery company to deliver the part within a certain time period;
company failed to deliver on time; as a result, mill could not operate and
they lost a considerable amount of business; the court held that delivery
company could not be liable for the loss in business because they did
not know of the part’s importance not could they have foreseen that
their breach would result in stoppage of the mill.
(i) The court holds that a breaching party will only be liable for
damages which: 1) fairly and reasonably arise naturally OR 2)
such as may reasonably be supposed to have been in the
contemplation of both parties, when the K was made, as the
probable result of the breach.
(ii) UCC 2-715: Consequential damages (if they flow, “had reason to
know”) – number 2 above.
(iii) This includes both actual knowledge and what P should have
known.
(iv)This kind of ruling is a departure from the principle that an injured
party should be compensated for the damages they incur.
(v) Why should it matter whether or not the delivery co. knew about the
importance of the part? Well, if they didn’t know about it you could
not say that they agreed to be liable for the loss in business because
usually that would not be the result of a delay in delivering a part.
(vi)There must be some damages for the late delivery, but it shouldn’t be
this extensive if he did not know of the special circumstances.
(vii) What might be another way the delivery company could deal with
this problem? They could specify in the K what risks or liability they
are willing to assume and what liability they are not willing to assume.
(b) Ex. Airlines: Bumping passengers; whatever liability an airline decides to
take on in regard to “bumping passengers” from flights is going to be
reflected in the overall prices of airline travel. If they are liable for all
damages, such as loss in business opportunities etc., the price is going to
soar. If they are only responsible for a limited amount of damages, the
price will be lower. Thus, as a matter of public policy, it may be better to
limit damages to those that are foreseeable.
(c) Mick Jagger Hypo: a group of guys come into your music store, buy
instruments and request that they be delivered Saturday to Arco Arena;
you fail to deliver; the guys turn out to be Mick Jagger and his bandmates
and they cannot perform because of your failure of delivery; they want to
sue for $1 million. If you really didn’t recognize the guys, it could be
argued that you reasonably could not have known the effects of your
failure to deliver and you will not be held to such extensive damages. IS
IT REASONABLE BEHAVIOR TO PUT THE MILLION DOLLAR
FATE ON THIS?
(i) IS IT FORESEEABLE THAT THE DELIVERY MIGHT NOT
COME THROUGH – if so important, he must make sure.
(ii) WHEN THE DELIVERY DOESN’T COME – he must mitigate
damages.
(d) Kenford Co. v. County of Erie: court held that County was not liable
for the failure of surrounding property values to rise when they
breached their K with Kenford to build a stadium on property that
they had donated (with the condition that the stadium would be built
AND promised to make K with Corp to run it) for that purpose.
(i) The court seems to say that the County is not liable because there was
nothing in the K stating that they were liable for the failure for
property values to increase.
(ii) But, that is not the issue. The issue is whether or not they should have
known or could have anticipated that their breach may cause the
failure of property values to rise and hurt the plaintiff’s expectation.
Clearly, they could have anticipated that result.
(iii) Prof thinks they decided this way because the court was
uncomfortable with the large amount of damages that the County
would be liable for.
3) Certainty
(a) Must prove with reasonable certainty what the foreseeable damages
actually would be.
(b) Ex: scam essay contest entrants to win house – what should cheated
contestants get?
(i) Restitution Damages: $200 entry fee
(ii) Expectation Damages: expectation of each entrant was the $ value of
each entry…if the house sold for $1.2mil and 1.2million people
entered, each is only worth $1. Each contestant believes he is going to
win.
(c) Fera v. Village Plaza, Inc.: (shop forced to move to different spot in mall)
P must show reasonable certainty of damages…more than preponderance
of evidence but not certainty.
(d) Contemporary Mission, Inc. v. Famous Music Corp., [542]: record
company sold rights to album…new co. dumped the singing priests. What
expectation damages for the priests?
(i) Singing Priests trend line? An album that has done x in the past is
likely to do y in the future as demonstrated from comparable album
starts. Music company tried to say that how an album performs is
completely uncertain, but the court rejects that contention and allows
evidence from a statistical analysis of the trend.
(ii) Spin-off Opportunities too speculative: priests tried to argue that
concerts and future albums were foreseeable, but court found these
damages were too speculative.
(e) Liquidated/Stipulated Damage Clauses: Fixing Foreseeability & Certainty
(i) Define damages up front in the contract
(ii) Liquidated Damages: reasonable estimate of damages in event of brief
(iii) Penalty: damages which are not based on expectations
(iv)Wasserman’s Inc. v. Township of Middletown, [543]: liquidated
damages clause called for 25% of avg. annual gross receipts;
Township said that the gross receipts are not liquidated damages but a
penalty; expectation damages would have been net income for the
years left in the lease.
1. Problem: harm is hard to measure… are the stipulated damages
a good faith reasonable estimate of future damages?
2. Court thinks that the harm isn’t hard to measure in this case –
the store’s profits were dismal…hugely different than their
gross receipts.
3. Lesson learned – say why the harm is hard to calculate when
you write the stipulated damages clause.

V) FINDING THE LAW OF THE CONTRACT


A) Discovering one’s rights and duties under a specific contract, not his rights and
duties under the law of contract.
B) Parol Evidence Rule
1) Governs whether or not evidence other than the written contract can be
presented in determining the duties and obligations of the contract.
2) Can be both writings as well as oral evidence.
3) Generally, we disfavor the introduction of any additional evidence and we like
to stick to looking at the written contract.
4) Why should we have this rule?
(i) Judicial Economy: fewer things are discussed and fewer things need to
be analyzed and we can get to a conclusion faster; some would argue,
however, that the writings and oral agreements being left out are
important and “real.”
(ii) We need to decide what is in the contract and the best way of doing
that is to look at the written agreement itself.
5) Hypo: Residential Real Estate Contract: “buyer acknowledges that the buyer
has made at least a casual inspection of the Property. Buyer accepts the
property for purchase “as is.” Two days before K is signed, Buyer faxes Seller
and tells her that he is rethinking his willingness to pay for the house because
it needs a new roof; Seller says I will have the roof fixed if you will change
your mind.
(i) This fax seems to add a term, but that term is not in the written K.
(ii) Can the fax be introduced into evidence under the parol evidence
rule???
6) Gianni v. R. Russell & Co.: G and R signed a contract which prohibited G
from selling tobacco in the building; G contended that there was an oral
agreement that R would give him exclusive rights to sell soda if he signed the
K; R let another party sell soda; the court held that Gianni could not
introduce evidence of an oral agreement from Russell giving G exclusive
rights to sell soda in the building.
(i) When you look at what would follow naturally from an agreement, the
exclusive rights to sell soda would have been included if there really
was an agreement to stop selling tobacco in return for exclusive selling
rights.
7) Bollinger v. Central Penn Quarry Stripping and Construction Co.: the court
allowed evidence of an oral agreement between B and Central Penn to
remove topsoil before digging and replacing topsoil after digging was
complete.
(i) There was sufficient evidence to show that there was an oral
agreement: the construction company began by removing the topsoil
as agreed and then later ceased doing so. Also, there was evidence
of a similar agreement and activity with a neighbor.
8) There really is no concrete rule for parol evidence. You have to look at the
broad picture; you begin with the written contract itself and then, under some
exceptions, other evidence can be introduced to show that there was a
legitimate oral understanding or prior agreement.
9) Step 1: Can extrinsic evidence be used to contradict terms of the agreement?
(i) Ask: have parties adopted a writing as the final expression of one or
more terms?
(ii) If so, agreement is “integrated,” meaning parol evidence may not be
admitted to contradict the terms in the agreement.
(iii) See Restatement 2d 209, 213(1), 215.
10) Step 2: If integrated, can extrinsic evidence be used to add terms to the
agreement?
(i) Ask: did parties intend the integrated agreement to be the complete
and exclusive statement of the terms? It is very hard to say that an
agreement is “complete and exclusive.”
(ii) “A contract always has all the terms, except the terms that are not
there.”
(iii) If so, agreement is a “complete integration,” meaning parol
evidence may not be admitted to add any terms.
(iv)See Rest. 2d 210, 213(2), 216
C) Interpreting Contract Language – are you agreeing to what you think, or put
down (are their limited sets of meanings).
1) Ambiguous Syntax – “Between thought and expression there lies a lifetime.”
(i) Ex: “All domestic water piping and rainwater piping installed above
finished ceilings shall be insulated.” Contractor interprets that all
domestic water & rainwater piping above finished ceilings shall be
insulated. Homeowner interprets and should have drafted it this way;
“All domestic water piping, and rainwater piping installed above
finished ceilings, shall be insulated.”
2) Frigaliment Importing Co. v. B.N.S. International Sales Corp.: “what did they
mean by ‘chicken’ in the contract?” They meant different things – how does
the court define chicken for the contract?
(i) Contract Itself
1. Size of Chicken: smaller birds had to be young chickens…how
does chicken mean different things in two different places in
the same contract? Court sees it as relevant, not controlling.
2. Price of “chicken”: D argued that $.33/lb could have been for
broilers because they were selling for $.35 to $.37. P argued
that $.33/lb was low, but that’s why they bought…it was a
good deal.
3. “Govt. Inspected Chicken” – suggestive of FDA definition but
not conclusive.
(ii) Parties Actions & Communications
1. Communications indicated that P, when questioned by D,
specified the more general usage of “chicken.” Buyer
complained about the type of chicken initially but then
acquiesced. (Watch out for parol evidence rule – court doesn’t
run aground of it here because P isn’t trying to “change” the
term, but is bringing in evidence as to the parties interpretation
of the term. More fundamentally, the cables were subsequent
to the contract – parol evidence rule only excludes prior
negotiations/agreements.)
(iii) “Objective Definitions”
1. Industry Standard: conflicting testimony & most who said
“chicken” = broiler also said they covered themselves by
specifying. Court would apply the jargon if P can prove the
industry standard is so pervasive that even a non-industry
expert would know. If D is new to the industry, P must prove
that D actually knew the terms accepted usage.
2. Govt. Standards: became very important that govt. standards
were referenced.
3. Dictionary
3) Raffles v. Wichelhaus: P and D had different understandings of which ship
the goods were supposed to be transported on; as such, the goods came in
later than later than expected; the court held that since there was no
agreement as to what ship was meant by “Peerless” there was no mutual
assent and thus no K was formed that could be breached.
(i) The weird thing about this case is that it doesn’t have an opinion; it
just tells you the arguments on both sides and then they just give a
judgment without explaining it.
(ii) Milward argues that the K only says that a ship called “Peerless” has
to deliver the cotton; it doesn’t matter what ship it is, as long as its
name is Peerless.
(iii) The defendants are arguing that they meant a specific ship named
Peerless that was not the same ship as thought of by the plaintiffs;
because they did not have a similar understanding to the term, there
was no real agreement to begin with.
4) Oswald v. Allen: A had two coin collections, one Swiss Coin Collection and
one rare coin collection; the rare coin collection had some Swiss coins; O
said he wanted to buy all her Swiss coins; A thought this meant only the Swiss
Coin Collection; since there is no reasonable way of determining exactly
what items were contracted for… there is no K.
5) PG&E v. G.W. Thomas Draynage & Rigging Co.: Thomas had signed a
contract saying they will indemnify P for ALL loss, damage, injury, caused by
T’s work; P thought this included damage to their property; T thought this
only included damage to the property of third parties; the CA Supreme
Court allowed T’s extrinsic evidence to be admissible because it spoke
directly to how a term was to be interpreted; extrinsic evidence is
admissible where it is relevant to a “reasonably susceptible” meaning of a
word or term in the K.
(i) The lower court adopted a “plain meaning” rule which effectively
prevents the defendant from introducing any extrinsic evidence that
would show that they only meant to indemnify themselves for injuries
to third parties not for injuries to the plaintiff.
(ii) SC reversed this holding and allowed the evidence.
(iii) They reason that “true meaning” is not contained in the plain
meaning of a word; often times the words to not capture the true
intention of the parties;
(iv)Contractual obligations are created by intentions not by “magic
words.”
(v) Distinguish: this is not adding or contradicting a term in the K, it is
helping to interpret a term that is in the K.
(vi)Thus, the trial court must hear the evidence that may point to a
contradictory interpretation of the words used in the contract.
(vii) Traynor expresses a very postmodern interpretation of language in
PG&E. He basically says that the language must be considered in
terms of its context, the circumstances under which the agreement
was formed, and the relative understanding and education of not
only the person writing it but the person hearing it.
D) Filling Gaps
1) There are areas in which there is simply no understanding of a situation (as
seen above), but more often it is the case that there is an agreement and that
there is just some misunderstanding over a small point. So, how does the court
fill in the gaps? We usually do this by examining “best efforts” and “good
faith.”
2) Example: In the Real World, MTV made contracts with people on the show
and paid them for their work on the show. However, there was never anything
in the K that said anything about syndication rights. Now you can see the Real
World re-runs on late night television shows; MTV is making money off of
syndication, but the stars of the show are not entitled to any of that money. In
most regular sitcoms there is always a provision about syndication rights.
3) Dalton v. Education Testing Service (ETS): D took the SAT twice and
improved by 410 points; D had signed a statement which gave ETS the right
to cancel any test score “if there is reason to question its validity”; the
agreement offered D five options including the ability to offer additional
information in explanation of his score; D offered additional information;
ETS continued to question the score; the court required ETS to fully
consider the additional information in good faith, otherwise they would
be in breach of the K; however, ETS is not required to release the score.
(i) There was a provision which said that if ETS thinks that a score in
invalid they can cancel the score, but the test-taker has several options:
1) provide additional information; 2) take a free retest; 3) cancel the
score and refund all fees; 4) third party review by institution receiving
score; OR 5) arbitration.
(ii) Dalton is claiming that ETS must show good faith and fair dealing and
secondly they must consider the additional information that he sent in.
(iii) However, the K did not say anything about “considering the
information” it just says that test-taker can send in more information.
(iv)There is this idea that in all contracts there is an implied duty of good
faith and fair dealing, which requires parties to act in good faith not
only in its express terms of the K, but in the spirit of the K.
(v) What is the purpose of sending in additional information, if not for
ETS to fairly consider it and use it as evidence in making their final
decision about cancellation. There is no expectation value in just
sending in the additional info, the value is in someone reading that
information before making a final decision. P. 616
4) Burger King Corp. v. Weaver: BK gave W two franchises; BK then authorized
another franchise to third party in the same area; W considered this a breach
of his franchise agreement because it violated good faith and fair dealing; the
court refused to imply an “exclusivity” clause in the franchise agreement
(where there was no such express term) based upon BK’s duty to perform
in good faith.
(i) What is the gap in this case? There is no clause in the franchise
agreement which prevents Burger King from allowing competing
franchises in the area.
(ii) The Court says that Florida law will not allow for Weaver to win
because such a holding would add an “exclusivity” clause to the
contract that was not in the express language of the K. Burger King
has fulfilled all of the express terms in good faith.
(iii) We can distinguish this case from Dalton because in Dalton the
express term seemed to imply (in fact, you could argue it is a logical
necessity) consideration of the additional information. The Option to
Give Evidence in Dalton doesn’t even make sense unless ETS is going
to give the information full consideration. Whereas, here none of the
express terms seem to imply an exclusivity clause as Weaver claims.
Weaver is really asking the court to add a term to the contract that was
never there.
5) Eastern v. Gulf: see additional facts above; Eastern was fueling at the
cheapest Gulf stations possible and not fueling where the prices were high;
this was an industry practice called “fuel freighting.” Eastern’s use of fuel
freighting was not an act in bad faith that would constitute a breach of K
because it was an accepted industry practice.
(i) It is somewhat like Burger King in that you have a business practice
which is not expressly mentioned in the K.
(ii) Gulf is saying that this is a bad faith practice even though its
prohibition is not expressly stated in the K; it is implied by good faith
that they will not carry on this practice.
(iii) The court looks at what is called the “course of dealing” between
Gulf and Eastern under this K and then they look at industry practice
and they decide that this is an acceptable practice which has been
utilized in the past and is followed by everyone else in the industry.
These are fairly objective measures for showing “good faith.”
(iv)Industry practice is similar to trade usage (which we saw in the
chickens case).
6) Ex. NFL Contract: in the sole judgment of the club, if the player’s skill is
unsatisfactory, the team can cut the player.
(i) This seems to take good faith out of the contract because the owner
can simply reject the player for any reason they want. Maybe they will
decided that one player is great but he is expensive and the second
team guy is almost as good but is cheaper, so they let the guy go and
use the second stringer. They hide this motivation by saying that the
great player is not as good as the second team player.
(ii) There is not much room for the player to argue because the contract
stipulates that the owner has sole discretion as to what is satisfactory
and what is not.
7) Market Street Associates v. Frey: JC Penney and GE had formed an
agreement; JC Penney sold its rights under the K to Market Street; GE
promised to finance improvements that JC Penney wanted to make to the
land; if the financing negotiations were to break down, Penney could buy the
land at below market price; MS sends GE a letter requesting financing for
improvements; the letter mentions nothing about Section 34 which allowed
them to buy the land; GE refused to finance because it was below their
minimum; MS exercised its right to buy the land under paragraph 34; if MS
believed that GE knew about Section 34, it was not bad faith to exclude
reference to that Section in their letter asking for financing.
(i) The Court reasons that the mere fact that Market Street did not
mention paragraph 34 is not concrete evidence that they did not act in
good faith; GE is a sophisticated party, they were original parties to
the agreement.
(ii) The Court says that there needs to be more discovery to find out MS
state of mind. If MS believed that GE knew about the provision, then
they should be allowed to buy the land. But, if MS was deliberately
trying to take advantage, then they should be barred from utilizing the
provision.
(iii) The duty of good faith is not necessarily a duty to candor; MS
doesn’t have to be completely upfront about everything, especially
when dealing with a sophisticated party that should know better.

(iv) 672
VI)PERFORMANCE & BREACH
A) What constitutes fulfilling my duties under the K? What constitutes not fulfilling
my duties under the K?
B) Conditions: (Restatement 2d Sec. 224) a condition is an event, not certain to
occur, which must occur, unless its nonoccurrence is excused, before performance
under a contact becomes due.
1) Effects of Conditions
(i) NFL Contract: if play is unsatisfactory, then team can cancel the
contract. Players continued employment is conditional upon the player
staying in shape.
(ii) Luttinger v. Rosen: P promised to buy house (on the condition that
they acquire a mortgage for $45,000 for 20 years at no more than
8.5%); P promised to use due diligence in getting mortgage; D
promised to sell and to return deposit if deal falls through; P applied
to one bank and failed to get the mortgage at the conditional rate;
they asked for the return of deposit; D refused; the court held that
the condition precedent (acquiring mortgage at the desired rate)
had not been met and thus Luttinger was not required to go
through with buying the house and was entitled to a return of
deposit.
a. Why is looking at only 1 bank considered due diligence? P had
used a knowledgeable lawyer who knew the environment and
he chose the bank with the best possible chance of offering the
desired rate. If this institution would not lend at the desired
rate, no respectable lending institution would.
b. What about the fact that the sellers offered to make up the
difference? The court says that the K clearly does not say that
they can make up the interest and furthermore, it really
complicates the situation when someone promises to cover the
difference in interest. How do you determine the tax on the
interest, etc? You cannot force the P’s to enter into a
complicated lending scheme if they don’t want to.
(iii) Internatio-Rotterdam v. River Brand Rice Mills, Inc.: D, rice
processor, entered into an agreement with P, exporter, to sell rice; the
agreement was to deliver the rice to Lake Charles or Houston “FAS”
in December of 1952 w/ two weeks notice from P; P failed to give
delivery instructions two weeks before the end of December; the court
held that December delivery was of the essence and IR’s failure to
issue delivery instructions on or before Dec. 17 was a condition
precedent to RB’s performance; the nonoccurrence of that
condition entitled RB to rescind or to treat is contractual
obligations as discharged.
a. “FAS Lake Charles and/or Houston.” FAS means “free
alongside ship”; we will bring it to your ship at Lake Charles or
Houston and then it is yours.
b. Where is the condition here? Delivery must be in December,
and actual delivery cannot be in December unless Internatio
gives notice on December 17, giving River Brand two weeks
notice for delivery.
c. Why does the court determine that the K specifically called for
delivery in December? Why was December of the essence?
i. December is a peak month for selling rice.
ii. Prices will change
iii. River Brand knew that it was going to be operating at
capacity and they would need strict schedules for
getting rid of rice.
d. Another way of looking at this is that Internatio breached the
K. They could argue that by not giving sufficient notice, they
were in breach of K. However, all RB is saying is that they
don’t have to perform.
2) Problems of Interpretation
(i) Duty v. Condition: an occurrence can be a duty required by the
agreement, a condition, or it can be both.
a. Ex. Suppose Cargo Owner has Ship Owner undertake a duty to
sail with the next wind and makes sailing with the next wind a
condition of Cargo Owner’s Duty to pay a 10% premium.
Sailing with the next wind will discharge Ship Owner’s duty
and will satisfy the condition of Cargo Owner’s duty. If Ship
Owner delays sailing, however, Cargo Owner will be entitled
to damages cause by Ship Owner’s delay and in addition will
not have to pay the premium.
(ii) Peacock Construction v. Modern Air Conditions, Inc.: Modern signed
a subcontract in which Peacock agreed to pay Modern “within 30
days after… full payment by Owner”; Owner has not paid, and P
asserts that payment by Owner is a condition of P’s payment to M; the
court says the Peacock must pay because as a matter of policy
courts do not recognize a condition precedent to the general
contractors duty to pay the subcontractors, unless it is specifically
stated in the K.
a. In most cases, the intent of the general contractor and the
subcontractor is that payment by Owner will not he a condition
precedent to the contractor’s duty to pay a subcontractor.
b. The idea is that it is the small subcontractor who is going to get
hurt worse by incurring the loss; much more so than the
general contractor
c. Also, the general contractor has a direct relationship with the
Owner; whereas the subcontractor does not. Peacock assumed
the risk of Owner not being able to pay; Modern did not, they
only assumed the risk that Peacock may not be able to pay.
d. Subcontractor would not ordinarily assume the risk that he
Owner might not pay; if they knew that they were assuming
such a risk, they probably would not do the job.
e. Thus, Peacock’s duty to pay was not conditional upon payment
by Owner. Peacock had a duty to pay Modern and they failed
to do so.
(iii) Pg. 676, Note 3: a stipulation stating that “owner promises to pay
broker 3% commission, payable upon closing” is considered a
condition precedent and thus broker will not get the 3% unless the
owner gets to closing. How is this distinguishable from the previous
case? The broker has much more of a connection with the purchaser of
the house than the subcontractor has with the Owner. The broker goes
out and finds purchasers who will be good and draws up all the
paperwork between the purchaser and the owner. Yet, if the owner
doesn’t get to closing, he gets to keep the deposit and he doesn’t have
to pay the commission to the broker.
a. What about the fact that the owner got to keep the deposit? He
only got the deposit because of the labor that the broker put in.
The deposit was supposed to be protection against costs the
owner might incur, including the 3% commission he would
have to pay to the broker. Shouldn’t the broker be entitled to
some of that money? Owner is still going to sell his house and
in addition he gets the full amount of the deposit; meanwhile
the broker is hung out to dry.
b. Holmes: you can always imply a condition in a K, the
question is why do you imply it.
c. We sort of feel now, in our current economic place and the
position of the parties, that the broker should not get his
commission. Our notions of fairness are colored by the
economic environment and practicalities of our time.
(iv)Mattei v. Hopper and Gibson v. Cranage
a. In Mattei, the court said that “satisfactory” is limited by good
faith.
b. In Gibson, the court seems to say that “satisfactory” is not
limited at all. The defendant (buyer) was the only person who
had the right to decide whether or not the portrait was
“satisfactory.”
c. How do we reconcile this difference? In this particular case,
the “satisfactory” condition is only in the agreement because
Cranage had originally said that he didn’t want to buy the
portrait. Cranage only agreed to have the portrait drawn up
because Gibson agreed that he would not have to buy if it was
unsatisfactory.
d. Also, this was to be a portrait of Cranage’s deceased daughter;
thus, the factors involved in determining “satisfaction” are
inherently going to be subjective and personal, there can be no
real objective factors.
e. Furthermore, there is really no agreement here. G says he is
going to create the portrait, and C essentially says: “Go ahead,
but I’m not promising anything in return.” There is no real
consideration.
f. In commercial situations like Mattei it is very hard to imagine
such a situation arising. A builder is not going to build a
building with the condition that the buyer only has to pay if he
is completely satisfied in accordance to his own subjective
determination.
3) Mitigating Doctrines Regarding Conditions
(i) Prevention: one who prevents an occurrence of a condition of one’s
own duty may be precluded from later asserting the non-occurrence of
that condition.
(ii) Waiver: an obligor whose duty is conditional may promise to perform
despite the nonoccurrence of a condition. Such a promise is called a
waiver.
a. If the condition is waived before the time for the occurrence
and the party can retract the waiver and reinstate the condition,
unless the party has relied to such an extent that retraction
would be unjust. If they have relied, the retraction is barred by
estoppel.
b. If the condition is waived after the time for the occurrence of
the condition, the court may say that the party has elected to
make the duty unconditional. When there is a non-occurrence,
the party with the conditional duty could 1) treat their duty as
discharged; OR 2) disregard the non-occurrence and thus elect
to peform the duty without any condition.
(iii) McKenna v. Vernon: Vernon waived his right to make his
payments to McKenna conditional upon certificate of completion
by the architect when he continually paid McKenna without a
certificate being asked for.
C) Mitigating Doctrines Regarding Incomplete Performance: Substantial
Performance
1) Ex. Construction Ks: Owner has a duty to pay; Builder has a duty to build;
generally, the implied condition on O’s duty to pay is that B must build first.
The concept is that you have to work first before you are paid.
(i) We would say that “building the house” is both a duty and a condition.
B has a duty to build the house, and his building of the house is a
condition upon being paid.
(ii) “Paying” is simply a duty, it is not a condition of anything.
(iii) However, situations often arise where the B builds the house, but
not in accordance with the specs in the K, and O says that he does not
have to pay since B did not meet the condition to build according to
the specs of the K.
(iv)This seems unfair to not allow B to get any money for his labor, even
if his labor may be less than perfect. This is why we say that B does
not have to perform perfectly, he must perform substantially. B is only
in breach when he fails to substantially perform.
(v) B’s duty is to build according to the specs, but the condition which
warrants payment is “substantial performance.”
(vi)O must pay B the contracted price minus the value of the error.
2) Substantial performance is not as applicable to sale agreements because the
transfer of goods and money can happen pretty much simultaneously.
3) What constitutes “substantial performance”? Jacob & Youngs v. Kent: J&Y’s
building of the house without using the Reading pipe in all situations was
considered a “substantial performance”; thus, J&Y did not breach the K.
(i) If Kent had stipulated that the use of Reading pipe in constructing the
plumbing of the house was explicitly a condition of final payment,
then we might be able to say that there was not “substantial
performance” and therefore a breach of the K.
(ii) If there was no “substantial performance”, does Kent get a free house?
NO. The court would rule that the K was breached by J&Y and award
damages to Kent for his loss, but they will allow J&Y (via the doctrine
of unjust enrichment) to recover the value of their labor.
4) Plante v. Jacobs: P contracted to build a house for J for $26,765 in
accordance with specs; J paid P $20,000; a dispute broke out over the
placement of a wall; J refused to continue payment; P did not complete the
house; the court held that the K was substantially performed because the
“essential purpose” of the K was met: building a house.
(i) P’s failure to build exactly to the specs was not a breach because the
details of the house were not made the essence of the K; there were no
blueprints or detailed construction in the plan.
(ii) P’s failure to construct a wall maximizing space in the living room did
not effect market value of the house; requiring the destruction of the
wall and rebuilding would be an economic waste and is therefore
unjustified. P cannot recover the cost of rebuilding the wall.
(iii) In this case, the purpose of the K is to give the Jacobs a house, thus
the precise size of the living room and the kitchen are mere details and
are not deviations from the main purpose of the K. As a result, the
court holds that Plante did “substantially perform.”
(iv)J is entitled to the cost of minor repairs
(v) P is entitled to the contract price minus the damages caused the
defendant by incomplete performance
5) Perfect Tender Rule
(i) The general rule in commercial agreements (sale of goods) is that the
buyer does not have to accept goods unless the seller makes “perfect
tender.” Learned Hand: “there is no room in commercial Ks for the
doctrine of substantial performance.”
(ii) UCC upholds the rule, but softens it in several ways.
(iii) UCC 2-601: they buyer may reject the goods if the goods differ in
any respect to what was contracted for. Covers not only the quantity of
the goods but also the details of the shipment.
(iv)UCC 2-508: allows the seller to cure the defective tender before the
time for performance has expired.
(v) UCC 2-608: allows a buyer who has already accepted goods to revoke
that acceptance, only if the non-conformity substantially affects the
value of the goods to him.
(vi)UCC 2-612: allows a buyer to reject an installment and claim breach
of the whole K as long as the non-conformity substantially affects the
value (negatively).
(vii) The seller must cure with perfect tender.
6) Why don’t we allow substantial performance in sale of goods?
(i) With goods, if the sale does not go through, the seller can walk away
with the goods.
(ii) In a labor/working/construction context, if the K is not fulfilled, the
work and materials are already spent; you can’t get them back.
(iii) With goods, the forfeiture is much less harsh allowing for a harsher
tender rule.
D) Prospective Nonperformance: Anticipatory Repudiation
1) Anticipatory Repudiation
(i) Two parties make an agreement to perform a transaction in the future.
One party calls before the time of performance and makes an
announcement of an intent not to perform.
(ii) Five Questions as to the Consequences of Anticipatory Repudiation:
a. Are you released from the K?
b. Can you sue me now?
c. Can you wait until Tues?
d. Can I change my mind before Tues.?
2) Hochster v. De La Tour: P agreed to be a courier for D over the summer for
10 pounds per month, beginning June 1st; in May, D said he would not employ
P as a courier; the court held that D’s renunciation of employment was a
breach, and the P could bring suit immediately; P did not have to wait
until June 1 to bring suit.
(i) To force P to wait until June 1 to sue would be economically
inefficient.
(ii) P was the one wronged here and it makes sense to make it easier for
him rather than making it more convenient for the breaching party.
(iii) Making P wait means that he cannot enter into any other
employment until June 1st otherwise he would be in breach of the K.
(iv)Should the P wait around until the day of performance?
a. This basically allows the P to act as if the breach is not a
breach and there is no duty to mitigate damages prior to when
the performance is due.
b. Courts are split on this issue.
3) McCloskey & Co. v. Minweld Steel Co.: Minweld did not repudiate the K
when they told McCloskey that they were having difficulty obtaining the
necessary steel for the subcontracting job and asked for assistance.
(i) Rule: in order for there to be a repudiation, there must be an absolute
and unequivocal refusal to perform or a distinct and positive statement
of an inability to do so.
(ii) The Court finds that Minweld did not repudiate because they simply
said that they were having difficulty in procuring the steel necessary
for the job and that they needed McCloskey’s assistance (as the
general contractor). They didn’t say that they would not do the job or
that they would be unable to do so.

VII) FAILURE OF BASIC ASSUMPTIONS: MISTAKE, IMPRACTICABILITY


AND FRUSTRATION
A) Basically, legitimate excuses for getting out of Ks.
B) Mutual Mistake
1) Restatement 2d § 152, 154:
2) Stees v. Leonard: L contracted to build a building for S; the soil upon which it
was to be constructed was composed of quicksand; L tried to build twice, but
each time the building collapsed; L refused to perform; even though it was
more difficult than anticipated, L had a duty to complete performance
and thus he had to do whatever was necessary to meet his duty.
(i) Building required more expensive foundations than called for in the K
and draining of the land; L was required to do these things in order to
complete performance.
(ii) The defendants are arguing that they adhered to all the specifications
in the K, yet they were unable to construct the building and thus they
should be allowed out of the K.
(iii) Rule: If a man bind himself, by a positive, express K, to do an act
in itself possible, he must perform… unless prevented by an act of
God, the law, or other party to the K. No hardship, no unforeseen
hindrance, no difficulty short of absolute impossibility, will excuse
him.
(iv)The Court says: tough luck for the builder, he must do what he
contracted to do.
3) Renner v. Kehl: R entered into a K with Kehl to purchase leases on land in
Arizona; R was going to use the land to cultivate jojoba; both parties believed
that there was adequate water supply for cultivation; R paid a down payment;
there was not sufficient water; R brought suit for rescission; the K was
voidable because there was a mutual mistake as to a basic assumption of
the K; however, R is not entitled to consequential damages.
(i) Rule applied in this case is different: the K is voidable where there is a
“mutual mistake” of a “material fact that constitutes an essential part
of the K.”
(ii) They determine that the availability of water on the land to grow
jojoba was such a “mutual mistake of material fact.”
(iii) This is material because D knew that Renner wanted the land for
growing an agricultural crop. Water is essential to growing crops.
(iv)The Restatement places another condition upon whether or not a
mutual mistake will void a K: whether or not the risk was assigned to
one of the parties?
(v) The Court essentially writes a condition into the K: we will allow this
K to be enforceable only if there is adequate water to grow the crop.
(vi)R gets his money back and Kehl gets his land back; R will also get a
sum equal to the value of his enhancement to the land. However, he
cannot recover his development costs.
(vii) There is no wrongdoing here. We just void the K and act as if it
had never happened, giving each side back what it had to begin with;
there are no damages.
4) When is mistake an excuse? Rest. 2d Sec. 152 and 154
(i) A mutual mistake allows a party to void the K if:
a. it goes to basic assumption;
b. it has a material effect on agreed exchange of performances;
c. a party does not bear the risk of the mistake.
(ii) Who bears risk of mistake? A party bears the risk if:
a. it is specifically allocated to the party by the K;
b. party went forward knowing that he had limited knowledge;
c. reasonable for the court to allocate the risk to the party.
5) Application to Renner:
(i) Renner thought he had complete knowledge that there was quality
water beneath the property which would allow for him to farm jojoba.
(ii) However, is it reasonable to allocate the risk to him? This is Arizona,
shouldn’t he have made sure that there was adequate water before he
bought the property and wasn’t he in effect bearing the risk if it turned
out that there was not adequate water?
6) Is this a good rule?
(i) We want there to be mutual assent in Ks – there is no foundation if the
K was based upon a mistaken assumption – there was no meeting of
the minds.
(ii) A rule like this could help with the encouragement of making K’s;
under the old rule, a party would be bound by a K even in the presence
of a freak occurrence – maybe people would be less likely to enter into
a K.
7) Distinguish Watkins v. Carrig: the mistaken fact (whether or not there was
hard rock beneath the house) was not a basic assumption in the K. The owner
was not assuming that the ground was soft soil that would be easy to dig.
(i) Even if it were a basic assumption, the excavator went forward and
contracted for the job knowing that he had limited knowledge of the
situation.
C) Changed Circumstances: Impracticability of Performance
1) Taylor v. Caldwell: Defendants contracted out their theatre to P for a series
of grand concerts; before the first party, the music hall burned down; P sued
for breach of K; in Ks in which the performance depends on the continued
existence of a given person or thing, a condition is implied that the
impossibility of performance arising from the perishing of the person or
thing shall excuse the performance. The K is void, both parties are
excused.
(i) The court begins with the basic rule that if one contracts to do
something, the contractor is required to perform it even if it becomes
unexpectedly burdensome to carry out that performance.
(ii) The court looks at it (via a doctrinal analysis) and says that both
parties made an assumption that the music hall would exist when it
came time for performance. This was an implied condition in the K,
and as such this was not a “positive contract” to do a thing. It is a
conditional agreement: performance is only required if the thing exists
at the time for performance.
(iii) You need to look at this case in terms of allocation of risk: to
which party should the risk be allocated?
a. If you make the music hall owners bear the risk of the party,
you are going to make them bear the loss for every contract
that they made for leasing the hall.
b. However, if you make the plaintiffs and other lessees bear the
loss, then the losses are dispersed among more parties.
(iv)It’s not so much that it is “impossible” for the music hall owner to
perform (because he could still perform by paying damages to the
plaintiffs), it is more so that it is “impractical” for him to perform.
2) Foreseeability
(i) Foreseeability doesn’t tell us that much, because if it is foreseeable
both parties could have foreseen it and accounted for it in the K.
(ii) Rule: Foreseeability of risk does not necessarily prove its allocation.
Just because a risk is foreseeable doesn’t mean that one party assumes
the loss should that risk occur.
(iii) Ex. Prof. agrees to cater Joel’s wedding. Joel wants a special cake
that is only made by this one cake-man and he only takes orders from
commercial caterers. The wedding is on June 1. Prof. calls the cake
guy on May 27, and the cake guy says he cannot make it because he is
all booked up. Who should bear the risk? Probably the Prof. because
he could have called much earlier about the cake, there was more that
he could have done to perform even if it was not necessarily
foreseeable that the cake-man would be booked.
3) In Eastern v. Gulf, the court says that Gulf could have foreseen the energy
crisis and the dramatic change in oil prices via price controls. The prices could
have changed in their favor or in a way that was disadvantageous to them.
Gulf could have protected themselves in the K, but they postulated no such
provision. As such, they should bear the allocation of risk when the price
changes served to hurt them.
(i) Gulf’s claim of hardship giving rise to “commercial impracticability”
fails: 1) their costs were illusory because they included intra-company
profits; 2) Gulf knew the risks when they agreed to tie the price to
Platt’s postings. Thus, they must be bound.
(ii) Ultimately, we are asking who is the best party to dump the risk on?
(iii) What you should take away from these cases is what sort of things
courts consider in making this allocation.
D) Changed Circumstances: Frustration of Purpose
1) It is not that important of a doctrine in America; it is not as strong of an
argument as “mistake” and “impracticability.”
2) Frustration is about the idea that after the K is made, if the principal purpose is
substantially frustrated by an event whose non-occurrence was a basic
assumption of which the K was made, then I am excused.
(i) Unless the language of the K or the circumstances of the K show the
contrary.
3) It is like impracticability in the sense that it is about the effect of changed
circumstances after the K has been formed. By contrast, mistake is about a
mutual understanding at the time of the formation of the K.
4) Krell v. Henry: H wanted to see the coronation of Queen Victoria and so he
rented a bunch of rooms in a hotel; coronation was postponed; H refused to
pay for rooms;
(i) H is arguing that his purpose in renting the rooms was to see the
coronation, and now that purpose has been frustrated, so he shouldn’t
have to pay.
5) Hypo: A leases B a lot; B intends to use the lot for a car dealership; U.S.
enters WWII and begins steel rationing and develops restrictions on the sale of
new cars; B can’t get any cars to sell and cannot get his dealership started;
should B be excused and let out of the K because of frustration of purpose?
(i) Saying “I can’t use the lot for a car dealership” is not sufficient to
show a frustration of purpose.
(ii) The court just says that you could use the lot for something else; the
lot still has value, so it can still be used in a productive way.
(iii) If B could show that the lot is not good for anything but a car
dealership or that he himself is not capable of carrying out any other
business, then he may have a case.
(iv)There has to be more than just interrupted motivation. You can’t just
say “Well, I was intending to do this, and now I can’t.”
6) Swift Canadian Co. v. Banet: Swift makes K to sell lamb pelts to Keystone in
the U.S.; U.S. issued stricter regulations on the importation of lamb pelts;
Keystone attempts to get out of the K because their purpose has been
frustrated; the court held that Banet was still obligated to buy the pelts
because the term “F.O.B Toronto” implied that Banet had assumed the
risk of any complications in shipping.
(i) Swift’s sale of the pelts was “F.O.B. Toronto.” This means that once
the pelts are put on the train in Toronto, the goods are Keystone’s and
they are responsible for the risk from then on. But, the regulations
were passed before Swift put the pelts on the train.
(ii) Swift’s duty was to simply make the pelts available.
(iii) Thus, it seems that the parties intent was to allocate most of the
risk to Keystone in getting the lamb pelts across the border. Most of
the risk was put upon Keystone with the term “F.O.B. Toronto.”
(iv)If Keystone wanted to relieve itself of risk they should have written the
K differently, but they likely would have had to accept a higher price.
There are always uncertainties in getting good across a border, so if
Swift was going to accept the risk they were going to want more
money.
E) REMEMBER: the real issue here is allocation of risk: you look at the intentions,
circumstances, common customs, the economics, and then you decide which party
should bear the risk.

VIII) THIRD PARTY BENEFICIARIES


Lawrence v. Fox
• The Court struggles with finding a way for Lawrence to sue Fox because
fundamentally Lawrence was not in contractual privity with Holly and Fox even
though the Lawrence was a clear beneficiary of their contract.
• How does the court allow get around this problem? They say that Fox was, by
analogy, a trustee of Holly’s debt to Lawrence and as a trustee he was required to
pay the debt to Lawrence. Lawrence is a beneficiary of this quasi-trust. Under
trust law the beneficiary has the right to sue the trustee for mismanagement of the
trust. Thus, by analogy, Lawrence should be able to sue Fox
• They also make an agency argument: Holly was an agent of Lawrence. As an
agent, Holly distributed his responsibility to Fox who then became responsible to
Lawrence.

Seaver v. Ransom
• Beaman’s husband told her that when he died he would leave the Plaintiff the
difference between the value of the house and what she got in the will. The
husband failed to do this upon his death.
• The question now is can Seaver enforce the promise that Judge Beman made to
his wife?
• The court reasons that Seaver is the one for whose benefit the agreement was
made and she is the one who suffered from the breach. She is the only one who
has suffered from the breach.
• One thing that makes this case weaker is the fact that in Lawrence v. Fox there
was already a debt relationship between Lawrence and Holly, but here there is
real contractual relationship between Mrs. Beman and Seaver. The relationship
here is really just Mrs. Beman’s desire to give something to Seaver.
• Note that Mrs. Beman’s will is not on the board here, nor is Judge Beman’s will.
The only thing on the board is Judge’s promise to Mrs. Beman that he would
provide for Seaver.
• However, Judge Beman cannot say that he did not know that Seaver was to
receive the house or an equivalent value

Grigerik v. Sharpe
• In this case we get into whether or not the parties “intended” for the there to be a
third party beneficiary.
• G contracts to buy land for a higher price ($16,000) in consideration from Lang’s
undertaking to get the town’s approval for the tract as a building lot.
• L contacts Sharpe to design a septic system in order to get a building permit for
the lot; the county sanitarian approved the design and the plans, and G buys the
land; later, a new sanitarian determined that the lot was not suited to a septic
system and they rejected G’s building permit.
• Did L and S “intend” for G to be third party beneficiary of their agreement?
• The lower court says we only need to look at what L intended, we don’t need to
look at what both L and S would have intended. Thus, they find that G was a
third party beneficiary and they find in his favor.
• The Court says that you must look at the intent of both parties to the K in
determining whether or not a third party is a beneficiary of the K.
• “the fact that a person is a foreseeable beneficiary of a contract is not sufficient
for him to claim rights as a third party beneficiary.”
• Contracts 101: any duty that is being placed upon a party (including duties to
third parties) must be something that the parties agreed to knowingly and
voluntarily.

Septembertide Publishing v. Stein & Day, Inc.


• Stein & Day goes bankrupt and they can no longer pay the royalties to
Semtembertide. So Septembertide goes to New American Library, who had a K
with S&D, and tells them to pay the money to Septembertide instead.
• Realistically, what is going on here is that Stein & Day (after bankruptcy) owes
money to several different parties including Septembertide. Septembertide wants
to prevent the money from going to S&D because once it gets there it is going to
be divided among several creditors. Septembertide wants to cut in line and get the
money before it goes to the pot.
• The court seems okay with looking at the intent of the promisee, Stein & Day, in
determining whether or not Septembertide was a third party beneficiary.
• Timing: the two Ks were made within days of eachother and S&D had told NL
that they needed the money from them in order to pay royalties to Septembertide.
• The court does not say that NL’s intent does not matter at all. They just kind of
brush this over. Instead they reason that NL should have known that
Septembertide was a third party beneficiary. This seems to be a forseeability
argument that was specifically rejected in Gregarik.
• The most convincing way of showing NL’s intent is by making a forseeability
argument. There was an industry practice for an author to receive a percentage of
royalties from sublicense agreements, so NL should have know that
Septembertide was a third party beneficiary.
• The Restatement (Second) of Contracts, Sec. 302 supports this saying that you
look at the promisee’s intent.

Three Ways of Determining Third Party Beneficiaries


• Restatement: only the promisee must have the intent of there being a third party
beneficiary
• Grigerik: you have to look at the intentions of both parties to see if they both
intended for there to be a third party beneficiary.
• “Economically Desireable”: do the economics of the situation require that the
party be treated as a third party beneficiary. Maybe this is a better argument for
Septembertide. It is the author’s work and if author’s are not paid royalties, they
would stop producing their literary works.
Municipal Contracts and Public Services
• If one party contracts with a city to provide water works and the firefighters are
unable to put out a fire because the water works are not sufficient, the court held
that the owner of the burned building could not bring suit against the company
providing the water works.
• Also in another case, Thruway had a K with Chevron allowing Chevron the
ability to sell gas on the Thruway as long as they provided services to persons
broken down or in need on the Thruway. Chevron failed to do so, and P husband
had a heart attack trying to change his own tire.

The Court focuses a lot of foreseeability in these circumstances.


• In the Chevron case, you might forsee that you should help with breakdowns, but
Chevron could not forsee that it might lead to one’s death.
• It is not foreseeable that a lack in water pressure would lead to a building burning
down.

In another case, NYC eneterd into a K with ConEd such that ConEd would provide
electric power of a certain quality to its customer. There is a black out and riots take place
causing damage to persons and property. The court holds that the citizens of NYC are
third party beneficiaries, but ConEd is only liable for a limited amount of damages. They
are only liable for those damages which are reasonably foreseeable from the black out.

Vesting of Rights
Hypo: say A owes money to B. Then C says she will pay B for A if A will help her fix
her car. Later on, C and A make a new deal. A says, “You don’t have to pay B for me, if
you fix my house.” If B wants his money and comes after A and A says “he doesn’t have
it,” can B then bring suit against C as a third party beneficiary? This is the kind of
situation you have in Olson v. Etheridge.
• Olson says that you cannot get out of an agreement made for a third party
beneficiary; that is the old rule.
• The Court disagrees, saying that until one of three things happens, the
contracting parties can change their agreement as many ways as they want
and the rights will not vest.
• One an agreement is made for the benefit of a third party, the rights of the
third party don’t vest until there is a definite act by B, a reliance act upon
the agreement,
• Thus, if A and C remake their deal before B comes to collect, B cannot
enforce the agreement against C. He could still enforce the agreement
against A.

IX)ASSIGNMENT & DELEGATION


Assignment of (K) Rights
• Say B owes A $100. A owes money to her landlord, C. In this situation you
cannot say that C is a third party beneficiary. However, A could assign his right to
the $100 to C. Now B owes nothing to A, but he does owe $100 to C. If B does
not pay, C can sue him, but A cannot sue B. Likewise, if B does not pay, C cannot
sue A, he can only sue B to get the money.
• In contrast, in a third party beneficiary situation, in which A does something for B
in return for B’s promise to pay C $100, C could sue either B or A in getting his
money.
• How can A make an assignment to C? A writing is not doctrinally required, but
practically it is required. A has to actually transfer the right to C either by a
writing with appropriate language or by actually giving C the actual paper
recording the IOU.
• There may be a situation in which A’s assignment of his right to C may put B in a
more difficult place. Perhaps, the assignment to C makes it much more difficult
for B to meet his obligation. In such case, maybe B’s right/duties are changed?

In Septembertide, Bookcraft was trying to claim that SD had assigned their rights to
monies from NL to them. S was saying that, actually, they were the third party
beneficiary of those monies, so they should get the proceeds from NL.

Example: Pg. 896


• Fiberlast contracted with Counter to build a Radome.
• In order to obtain financing to build the Radome, F got financing from D.
• D made his financing conditional that any money F received from Counter would
be immediately paid back to D.
• This is not an assignment because the money passes through someone else’s
hands before D get the money.

Fiberlast → Drew

Counter

Herzog v. Irace
• The court says that there was a de facto assignment. Jones made a present transfer
of a future right when he wrote the “I request that payment be made directly from
settlement of a claim currently pending for an unrelated incident, to John
Herzog…”
• The lawyers were saying that they could not give the money to Herzog because
they were under an ethical obligation to do with the money as their client directs.
One thing they could have done was put all the money in an escrow account and
allow the court to decide how that money should be dispersed.
• This is a good example of a lawful assignment.

Assignment v. Delegation
• Assignment refers to a transfer of rights
• Delegation refers to a transfer of duties or obligations
• Delegation: if B owes a duty to A, B can delegate his duty to D. D now has a duty
to perform for A; however, if D fails to perform, B still is responsible for
performing the duty.

Questions that Arise in Delegation?


• Is the delegation really effective?
• If one is substituted to perform the duty, does the benefited party have to accept
performance from the substitute? Or can he demand performance by the original
party.

Ex. Pavarotti cannot substitute Michael Jackson in completing his duty to sing to airport
customers. It would not even be enough for Pavarotti to substitute another good opera
singer because this is a personal service contract. In such contracts, the benefited party
can refuse to receive performance from a substitute.

• Under a contract you have both rights and duties


• There may be some situations in which you want to assign both your rights and
duties.

A.C. Associates v. Metro Steel Industries, Inc.


• Walsh assigned “the contract” between Walsh and Metro to Presbyterian
Hosptial.
• The question is how to interpret the use of the word “the contract.”
• Many jurisdictions require one to specify whether they are assigning right or
delegating a duty or both. “I assign my K” will simply assign the rights under the
contract.
• Modern courts have begun to hold that “I assign my K” means an assignment of
rights and a delegation of duties.
• In this case Walsh has a right to Metro’s steel work. However, Walsh also has a
duty to Metro to pay them.
• NY followed the rule that “in the absence of an agreement to the contrary, the
assignment of a bilateral contract cannot be interpreted as a promise on the part of
the assignee to assumed the duties and liabilities imposed by the agreement upon
the assignor.”
• Thus, Presbyterian Hospital is not obligated to pay Metro.

Sally Beauty Co. v. Nexxus Products Co.


• Sally took over the Best company and Best ceased to exist; Nexxus had an
agreement with Best giving them exclusive rights to distribute Nexxus products
in Texas.
• The issue is whether or not Sally took over the rights and obligations of the
agreement with Nexxus when they bought out Best.
• Nexxus gave Best an exclusive right to distribute; however Best also owed
Nexxus a duty to use “best efforts” to sell Nexxus products. So can Best assign
their exclusive rights to Sally? Can Best delegate their “best efforts” duty to
Sally?
• The trial court held that the duties are not delegable because this is a personal
service based upon a personal relationship between Nexxus and Best.
• Sally says that this rule cannot be applied because that only applies when there is
a personal service; here there is only a agreement for selling shampoo.
• The Appellate Court says that it is more like a personal service because the duties
to Nexxus are more than just paying money, there is also a duty to use “best
efforts.” Sally is a wholly-owned subsidiary of Nexxus’ competitor, so there is no
way that they can be expected to use the same “best efforts” as Best would have.
N chose Best to perform because they personally trusted Best and that same trust
is not present in Sally.
• Dissent: If N really felt that Sally was not going to use “best efforts” they could
always sue Sally when their performance fails to meet expectations.
• Furthermore, the economics of the situation do not really allow for Sally to not
use “best efforts.” If Sally really tries to undersell Nexxus products, they would
only hurt their own business reputation. If Sally plays favorites, many powerful
competitors will not use them as a distributor. Professor thinks that this is an
assumption that may not necessarily be true; there has not been a sufficient
economic analysis to determine whether or not it would be beneficial for Sally
not to use “best efforts.”

Exam Stuff
• The exam is going to focus primarily on the second semester
• Format will be like last time: two essays

Parol Evidence
• If you have a written agreement that is integrated, then extrinsic evidence of prior
negotiations (written or oral) is not admissible to change the terms of the K.
• “Integration”: the final expression of the agreement. How do you know whether
or not the written document is the final expression?
• You can introduce evidence that calls into question the finality of that written
agreement. Ex. Bollinger case. The fact that the company had started by replacing
the topsoil suggests that the company believed that there was a previous oral
agreement
• Distinguish PG&E: there they are trying to interpret the meaning of “all” OR
what the parties meant by the word “all”; they are not trying to introduce evidence
of a prior agreement in order to vary a term in the written agreement.

Damages
• Formula A: LV – CA – LA + Other

Even though the court says they are providing expectation damages in Algernon it is
more like reliance.

Vous aimerez peut-être aussi